2ちゃんねる ■掲示板に戻る■ 全部 1- 最新50    

■ このスレッドは過去ログ倉庫に格納されています

不等式への招待 第10章

386 :132人目の素数さん:2020/04/08(水) 11:23:03 ID:pDfrzDrp.net
(1) x^4 + x^3 - 2x + 1 > 0,
(2) x^4 + x^3 - 2x + 6/7 > 0,

高校数学の質問スレPart404.051〜068

387 :132人目の素数さん:2020/04/08(水) 11:39:19 ID:7I0d5fbg.net
>>386
(1)は見た瞬間にグラフの概形が頭に描かれたわ。
訓練された不等式ヲタとは、そういうものだ。

388 :132人目の素数さん:2020/04/08(水) 11:46:58 ID:pDfrzDrp.net
(1) の方は
 (1/4)x^4 + x^3 - 2x + 1 = (xx/2 +x -1)^2,
だが・・・・

389 :132人目の素数さん:2020/04/08(水) 11:57:16 ID:7I0d5fbg.net
>>388
平方完成は瞬時にはできんかったわい。
3流不等式ヲタでスマン。

390 :132人目の素数さん:2020/04/08(水) 13:59:31 ID:7I0d5fbg.net
x,y>0に対して、
(x^x)*(y^y)*(Γ((x+y)/2))^2 ≦ Γ(x)*Γ(y)*((x+y)/2)^(x+y).
ここで、Γはガンマ関数

391 :132人目の素数さん:2020/04/08(水) 23:51:37 ID:pDfrzDrp.net
>>390
 f(x) = log{Γ(x)} - x・log(x),
 f '(x) = ψ(x) - log(x) - 1,
 f "(x) = ψ ' (x) - 1/x > 0,
ここに ψ(x) = Γ '(x)/Γ(x) ・・・・ digamma関数。

∴ f(x) は下に凸。

392 :132人目の素数さん:2020/04/08(水) 23:57:56 ID:pDfrzDrp.net
>>386
(2)
まず、高次の項を見て
(左辺)=(xx +x/2 -c)^2 +(2c-1/4)xx -(2-c)x + (6/7-cc),
とする。cは定数。

左辺は x = 0.607 の辺りで最小になるので |xx +x/2 -c| も小さいはず。
→ x=0.6 で xx +x/2 -c = 0 となるように c=0.66 とする。

(左辺)=(xx +x/2 -0.66)^2 + 1.07xx - 1.34x + 0.421542857
  ={(x-0.6)(x+1.1)}^2 + 1.07(x-67/107)^2 + 0.002010
  ≧ 0.002010

393 :132人目の素数さん:2020/04/10(金) 03:26:14 ID:IAsBrfBV.net
>>390

・E. Artin: "Entfuhrung in die Theorie der Gammafunktion",Hamburg (1931)

・高木貞治:「解析概論」改訂第三版, 岩波書店 (1961)
  第5章 §68. ガンマ函数

・E.アルチン「ガンマ関数入門」(はじめよう数学6), 日本評論社 (2002)
   p.126 2200円 上野健爾 [訳・解説]
 http://www.nippyo.co.jp/shop/book/1985.html

394 :132人目の素数さん:2020/04/10(金) 12:07:36.86 ID:HQzXTvXu.net
三角形の辺長 a,b,c および面積 S に対して、
√(aa+bb-4S) + √(aa+cc-4S) ≧ √(bb+cc-4S).

395 :132人目の素数さん:2020/04/11(土) 14:55:32 ID:jVXfLHUH.net
 BC = a, CA = b, AB = c としよう。
頂点Aから対辺BCに下した垂線(の延長線)上に、
 AD = BC = a
となる点Dをとると
 CD = √(aa+bb-2ab・sinC) = √(aa+bb-4S),
 BD = √(aa+cc-2ac・sinB) = √(aa+cc-4S),
で、どうする?

396 :132人目の素数さん:2020/04/18(土) 20:45:04 ID:/wfIVimW.net
ツイッターのとあるユーザーから出題

https://i.imgur.com/rLS41sF.jpg

397 :132人目の素数さん:2020/04/18(土) 22:43:06 ID:Mmg17QlQ.net
>>396
うむ、不等式信者が増えているようで何より。

398 :132人目の素数さん:2020/04/19(日) 05:23:49.74 ID:Cq2k8yf8.net
>>396
 Tenma Inequality Contest
次を示せ。
1. x,y,z≧0、x+y+z=1 のとき
  7/9 ≦ (xyz+1)/(xy+yz+zx+1) ≦ 1,

2. x,y,z≧0、 xyz=1 のとき
 (y/x + z/y + x/z) + (x/y + y/z + z/x) ≧ (x+y+z) + (1/x + 1/y + 1/z) ≧ 6,

3. x,y,z>0, x+y+z=1 のとき
 1/{x(y+z)} + 1/{y(z+x)} + 1/{z(x+y)} ≧ 27/2 ≧ 1/(x^4 + y^4 + z^4 + xyz),

4. x,y,z≧0 のとき
 3(x^3+y^3+z^3 +1) + 4(xy+yz+zx) ≧ 9xyz + 4(x+y+z),

399 :132人目の素数さん:2020/04/19(日) 05:49:57 ID:Cq2k8yf8.net
>>398
1.
 s = x+y+z
 t = xy+yz+zx
 u = xyz
とおく。
左)
 u + 1 - (7/9)(t+1) = u + s^3 - (7/9)s(t+ss)
 = (2s^3 -7st+9u)/9
 ≧ {(s^3 -4st+9u) + s(ss-3t)} ≧ 0,
 等号成立は x=y=z=1/3,
右)
 (分母) - (分子) = (xy+yz+zx) - xyz
 = (x+y+z)(xy+yz+zx) - xyz
 = (x+y)(x+z)(z+x)
 ≧ 0,
 等号成立は {x,y,z} = {0,0,1}

400 :132人目の素数さん:2020/04/19(日) 06:02:04.63 ID:Cq2k8yf8.net
>>398
2.
xyz = G^3 とすると AM-GMで
 x+y+z ≧ 3G, 1/x+1/y+1/z ≧ 3/G,
(左辺) = (x+y+z)(1/x+1/y+1/z) -3
 ≧ 3(x+y+z)/G + 3G(1/x+1/y+1/z) -12
 ≧ (x+y+z)/G + G(1/x+1/y+1/z),


なお、対称式でなくても AM-GMで
 (1/3)(x/y+x/y+y/z) ≧ x/G, etc.
巡回的にたすと
 x/y + y/z + z/x ≧ (x+y+z)/G,
同様にして
 (1/3)(x/y+y/z+y/z) ≧ G/z, etc.
巡回的にたすと
 x/y + y/z + z/x ≧ (1/x+1/y+1/z)G,
は出る。

文献[9] 佐藤(訳) 朝倉書店 (2013) p.26 演習問題1.75

401 :132人目の素数さん:2020/04/19(日) 06:22:15.43 ID:Cq2k8yf8.net
>>398
3.
 1/(x(y+z))= 1/(x(s-x))={1/x + 1/(s-x)}/s,
(左辺)≧{1/x + 1/y + 1/z + 1/(s-x)+ 1/(s-y)+ 1/(s-z)}/s
 ≧{9/(x+y+z)}+ 9/(3s-x-y-z)}/s   (← AM-HM)
 = 9/(ss)+ 9/(2ss)
 = 27/(2ss),

x^4 +y^4 +z^4 + xyz
 ≧(x+y+z)(x^3+y^3+z^3)/3 + xyz
 = s(s^3 -3st +3u)/3 + u
 =(s^3 -3st +6u)/3   (s=1)
 =(2/9)(s^3 -4st +9u)+(1/27)s(ss-3t)+(2/27)s^3
 ≧(2/27)s^3,
∴(右辺)≦ 27/(2s^3),

402 :132人目の素数さん:2020/04/19(日) 10:07:36.72 ID:Cq2k8yf8.net
>>398
4.
(左辺)-(右辺)= 3s(ss-3t)+4t -4s +3
 ={(3s^3 -4s +3)(ss-3t) + t(2s-3)^2}/ss
 ≧ 0,

3s^3 -4s +3 = 3(s+4/3)(s-2/3)^2 + 11/9 ≧ 11/9.

403 :132人目の素数さん:2020/04/21(火) 07:08:39 ID:VEDuEVHa.net
駿台の過去問らしい

https://i.imgur.com/FH292z6.jpg

404 :132人目の素数さん:2020/04/21(火) 08:05:27 ID:6SMLYdGW.net
>>403
〔問題5〕
nを2以上の整数とし、a_1, a_2, ・・・・, a_n を正の整数とする。
このとき、次の3つの条件をみたす正の整数 b_1, b_2, ・・・・, b_n が存在することを示せ。
(A) i = 1,2,・・・・,n に対して a_i≦b_i である。
(B) b_1, b_2, ・・・・, b_n をnで割った余りはすべて異なる。
(C) 不等式
  b_1 + b_2 + ・・・・+ b_n ≦ n((n-1)/2 +[(a_1+a_2+・・・・+a_n)/n]).
 が成り立つ。
 ただし、実数xに対してxを超えない最大の整数を[x]で表わす。

>>397
 そうかなぁ?

405 :132人目の素数さん:2020/04/28(火) 06:57:59 ID:A7QrgcmB.net
R^3上の(可測)集合A,Bに対して、
A+B:={x+y | x∈A,y∈B}とする.
このとき、
(A+Bの体積)^(1/3)≧(Aの体積)^(1/3)+(Bの体積)^(1/3)
を証明せよ.

406 :132人目の素数さん:2020/04/28(火) 07:02:02 ID:A7QrgcmB.net
>>405
ちなみに、A,Bを2次元平面上の図形として、
(A+Bの面積)^(1/2)≧(Aの面積)^(1/2)+(Bの面積)^(1/2)
となることは比較的簡単に証明出来ます

407 :132人目の素数さん:2020/05/03(日) 10:38:33 ID:u2nPgxPR.net
a,b,c>0, λ≧0
(a/b + b/c + c/a){a/(a+λb) + b/(b+λc) + c/(c+λa)} ≧ 9/(λ+1).

RMMかどこかで昔拾ったものだったか?

 ( ゚∀゚) プゥ
  ノヽノ) =3 'A`)ノ ヒャー
  くく   へヘノ

408 :132人目の素数さん:2020/05/04(月) 16:31:00 ID:jDRWX2Ph.net
3月の宿題で(1)のみ正解の数弱@shukudai_sujaku

昨年度の大学への数学(大数)での勝率は、

学コンBコースが 1/1 = 100% ,

宿題が 3/10 = 30% でした!

宿題の勝率が低すぎると思うので、

これからは一層精進していきたいです!

https://twitter.com/shukudai_sujaku
(deleted an unsolicited ad)

409 :132人目の素数さん:2020/05/05(火) 12:29:59 ID:b2IqdVzK.net
3月の宿題で(1)のみ正解の数弱@shukudai_sujaku

昨年度の大学への数学(大数)での勝率は、

学コンBコースが 1/1 = 100% ,

宿題が 3/10 = 30% でした!

宿題の勝率が低すぎると思うので、

これからは一層精進していきたいです!

https://twitter.com/shukudai_sujaku
(deleted an unsolicited ad)

410 :132人目の素数さん:2020/05/05(火) 17:26:38 ID:B6ZQqn9P.net
>>406
平面のときのも全然証明わかりません
証明もしくは出典教えてほしいです

411 :132人目の素数さん:2020/05/05(火) 23:55:57 ID:2A4Z2g/l.net
>>410
ヒントを言うとまずはA,B共に長方形の場合で示してください
ある有名不等式になります

どうしても答えが知りたければ
「Brunn–Minkowskiの不等式」でググってください

412 :132人目の素数さん:2020/05/06(水) 00:23:27 ID:1eDWV5m3.net
>>411
ありがとう
調べたらかなり有名みたいだね

もちろん最初に同じ向きの長方形の場合を考えてみたけど、そこから一般の場合にどう持っていくのかが謎すぎた
和をとった後の図形がどれくらい小さくなるかは元の図形の大域的な様子が必要そうに思えてさ
wikiぱっと見た感じでは凸包をとってよくて、さらに凸図形の場合は表面積の情報で抑えられるという感じなのかな

413 :132人目の素数さん:2020/05/06(水) 00:35:55 ID:sAc0zF0B.net
>>412
測度論で長方形から一般の可測集合に主張を拡張したい場合は「Dynkin's π-λ theorem」という便利な定理があります

414 :132人目の素数さん:2020/05/06(水) 00:45:25 ID:sAc0zF0B.net
適当にパラメータ付けした図形にBrunn–Minkowskiの不等式を適応すれば
大量に不等式を生成することが出来ます

415 :132人目の素数さん:2020/05/08(金) 01:13:17 ID:C8tI9fQ4.net
ツイッターから拾ったのでどうぞ

https://i.imgur.com/aYWdlZa.png

https://i.imgur.com/kMzfG2o.png

https://i.imgur.com/K7zRV0s.png

https://i.imgur.com/v6d3y6p.png


https://i.imgur.com/RJqT4Pu.jpg


https://i.imgur.com/zocnJSH.png

416 :132人目の素数さん:2020/05/08(金) 11:46:01 ID:WmDpVhCu.net
3月の宿題で(1)のみ正解の数弱@shukudai_sujaku

昨年度の大学への数学(大数)での勝率は、

学コンBコースが 1/1 = 100% ,

宿題が 3/10 = 30% でした!

宿題の勝率が低すぎると思うので、

これからは一層精進していきたいです!

https://twitter.com/shukudai_sujaku
(deleted an unsolicited ad)

417 :132人目の素数さん:2020/05/08(金) 20:28:34 ID:gGrwQusG.net
>>415

a,b,c>0 のとき、
 30sst ≦ 7s^4 + 9tt + 54su,
を示せ。
ここに s = a+b+c, t = ab+bc+ca, u = abc とおいた。


実数列{a_i},{b_j},{c_k}がある。以下の不等式が成り立つことを証明せよ。
(Σaa)(Σbc)^2 +(Σbb)(Σca)^2 +(Σcc)(Σab)^2 ≦(Σaa)(Σbb)(Σcc)+ 2(Σab)(Σbc)(Σca),


x1,x2,・・・,xn を実数とし、x=(x1,x2,・・・・,xn)と書く。
f(x)と g(x)は対称式である。
f(x)≦ g(x)が成り立ち、
等号成立が x = x_p =(y1,y2,・・・・,yn)に限るとしても
 f(x)≦ g(x_p)
が成り立たない例があることを示せ。


0 < x,y,z のとき、
 x/y + y/z + z/x ≧ (x+y+z)^2 /(xy+yz+zx),
を示せ。

5(改)
 1 < log([(e-2)x+2]/[(e-2)(x-1)+2]) + x・log(1+1/x)< f(x), (x>1)
ここに f(x)={x・log(x)+ log(x+1)- log(2Γ(x+1))}/(x-1),
 lim[x→1]f(x)= γ + 1/2,
 lim[x→∞]f(x)= 1,


∀x,y ∈[0,π/2)
 sin(x+y)+ tan(x)tan(y)≦ √{1+tan(x)^2}・√{1+tan(y)^2},

418 :132人目の素数さん:2020/05/08(金) 20:45:55 ID:gGrwQusG.net

 (右辺)= 7s^4 + 9tt + 54su
 = 30sst +(ss-3t)^2 + 6s(s^3 -4st+9u)
 = 30sst +(F_0)^2 + 6s・F_1
 ≧ 30sst,
 F_0 = ss -3t ≧ 0, F_1 = s^3 -4st +9u ≧ 0. (シューア)


 f(x)= Σ[i=1,n] (x_i)^2, g(x)= 2Σ[i=1,n] (x_i)^2,
 g(x_p)= g(o)= 0.
何か勘違いしてるかな?


コーシーで。
(分数形のものは Engel型のCauchy とか、Arthur Engelの最小原理と呼ぶらしい。)
文献[9] 佐藤(訳) 朝倉書店 (2013) p.44-45


 (左辺)^2 ≦{1 +|tan(x)tan(y)|}^2
 ≦ √{[1 +|tan(x)tan(y)|]^2 +[tan(x)±tan(y)]^2}/2
 ={1+tan(x)^2} {1+tan(y)^2},
或いは同じことだが
 (左辺)^2 ≦(1 +|tan(x)tan(y)|)^2
 ={cos(x)cos(y)± sin(x)sin(y)}^2 /{cos(x)cos(y)}^2
 = cos(x干y)^2 /{cos(x)cos(y)}^2  (←複号は適当な方をとる)
 ≦ 1/{cos(x)cos(y)}^2,

419 :132人目の素数さん:2020/05/08(金) 23:59:46 ID:gGrwQusG.net

 Σaa = AA, Σbb = BB, Σcc = CC,
 Σab = AB cosγ, Σbc = BC cosα, Σca = CA cosβ,
とおく。(0≦α,β,γ≦π)
球面三角不等式
 |α-β|≦ γ ≦ α+β,
より
 cos(α-β)≧ cosγ ≧ cos(α+β),
|cosγ - cosα・cosβ|≦|sinα・sinβ|,
2乗して整理すると
(cosα)^2 +(cosβ)^2 +(cosγ)^2 ≦ 1 + 2(cosα)(cosβ)(cosγ),
これに(ABC)^2 を掛ければ
 (Σaa)(Σbc)^2 +(Σbb)(Σca)^2 +(Σcc)(Σab)^2 ≦(Σaa)(Σbb)(Σcc)+ 2(Σab)(Σbc)(Σca).

420 :132人目の素数さん:2020/05/09(土) 06:14:35 ID:pHr5kdzK.net

x≒1 では
x・log(x)+ log(x+1)- log(2)=(3/2)(x-1)+(3/8)(x-1)^2 -(1/8)(x-1)^3 + ・・・
log{Γ(x+1)}=(1-γ)(x-1)+(ππ-6)/12・(x-1)^2 - 0.0673523(x-1)^3 + ・・・・
f(x)=(γ+1/2)+(7/8 - ππ/12)(x-1) - 0.0576477(x-1)^3 + ・・・・

x >>1 では
x・log(x)+ log(x+1)=(x+1)log(x)+ 1/x - 1/(2xx)+ 1/(3x^3)- ・・・・
log{2Γ(x+1)}=(x+1/2)log(x)-x +(1/2)log(8π)+ 1/(12x) - 1/(360x^3) + ・・・
f(x) ={x +(1/2)log(x)-(1/2)log(8π)+ 11/(12x)- 1/(2xx)+ 121/(360x^3)}/(x-1)
 → 1 (x→∞)

6(訂正)
 (左辺)^2 ≦{1 + |tan(x)tan(y)|}^2
 ≦{1 + |tan(x)tan(y)| })^2 +{ |tan(x)| - |tan(y)| }^2
 ={1 + tan(x)^2} {1 + tan(y)^2},

421 :132人目の素数さん:2020/05/09(土) 20:40:48 ID:pHr5kdzK.net
引き続く3つの奇数の和が54を超えないとき、
小さい数の最大値を求めよ。

The sum of three consecutive odd numbers is not greater than 54,
find the largest value of the smallest number.

數學999
http://www.youtube.com/watch?v=GTehFFxQCp8 02:42

422 :132人目の素数さん:2020/05/14(木) 01:49:21.16 ID:431XMp+q.net
n, k は2以上の整数、Aはn×nエルミート行列のとき、
(det A)^(1/n) ≦ [{(tr A)^k - tr(A^k)}/(n^k - n)]^(1/k).

423 :132人目の素数さん:2020/05/14(木) 08:31:45 ID:w+h9h8DE.net
>>419

      | (a・a) (a・b) (a・c) |
RHS - LHS =|(b・a) (b・b) (b・c) |  (Grammian)
      | (c・a) (c・b) (c・c) |

  | a |
 =|b|| a b c | ≧ 0,
  | c |

桑野耕一「ラグランジュ恒等式とは何か」
  数学セミナー、連載(2006年4月号〜)

なお、上式は(|a||b||c|)^2 以下になる。(Hadamardの不等式)

高木:「解析概論」改訂第三版、岩波書店(1961)
 第2章 §26.極大・極小 [例2] p.72〜75
分かスレ459.588, 594, 759 など。

424 :132人目の素数さん:2020/05/18(月) 18:07:02 ID:gzO4I8NK.net
ツイッターから拾った
https://i.imgur.com/A46WIGT.jpg

425 :132人目の素数さん:2020/05/18(月) 21:35:49 ID:0gqdjMjt.net
〔問題〕
x,y,zが非負実数全体を動くとき
 (x+y+z)^3 /(xyy+yzz+zxx+xyz)≧ 27/4,
を示せ。

例の問題...

426 :132人目の素数さん:2020/05/18(月) 21:49:34 ID:0gqdjMjt.net
>>425
(略証)
0 ≦ Min{x,y,z}= x としてもよい。
4(x+y+z)^3 - 27(xyy+yzz+zxx+xyz)
 = 9x(xx+yy+zz-xy-yz-zx) + (y+4z-5x)(x-2y+z)^2 ≧ 0,
等号成立は(x,y,z) = (1,1,1) (0,1,2) (1,2,0) (2,0,1)
[前スレ.014-020]
文献[8] 安藤:「不等式」数学書房 (2012)例題2.2.12(7)p.56
Inequalitybot [169]

(類題)
カナダMO-1995 A5
イギリスMO-2009 A4
Inequalitybot [61]

427 :132人目の素数さん:2020/05/18(月) 23:15:09.32 ID:0gqdjMjt.net
>>407

(1+λ)(a/b + b/c + c/a)≧(a/b + b/c + c/a)+ 3λ  (AM-GM)
 =(a+λb)/b +(b+λc)/c +(c+λa)/a,
∴ コーシーで
(1+λ)・LHS ≧{√(a/b)+ √(b/c)+ √(c/a)}^2
 ≧ 3^2       (AM-GM)
 = 9,

428 :132人目の素数さん:2020/05/19(火) 01:32:34 ID:G3uxUJZ9.net
>>426
巡回的なものを非巡回的な式に変形して示すの不思議
巡回的な変形でも示せるのか気になる

429 :132人目の素数さん:2020/05/19(火) 12:59:55 ID:C7hbQ2t7.net
>>419
>>423
2次形式
 f(x,y,z)=|xa + yb + zc|^2
が半正値だから、
RHS - LHS =(判別式)≧ 0,

430 :132人目の素数さん:2020/05/22(金) 07:21:06.00 ID:y+ggBWMl.net
>>421
(略解)
Let x be the smallest odd number.
x + (x+2) + (x+4) ≦ 54,
     3x + 6 ≦ 54,
       x ≦ 16,
       x ≦ 15, (x is odd number)
∴ The largest value of the smallest nember is 15.

431 :132人目の素数さん:2020/05/25(月) 19:54:27 ID:FyhIbn1M.net
a,b,c>0, a+b+c=1
36/(aab+bbc+cca) + 1/(abc) ≧ {24(aab+bbc+cca-6abc)^2}/abc + 343

432 :132人目の素数さん:2020/05/26(火) 22:40:14 ID:jFtHfHtW.net
a,b,c>0、G=(abc)^(1/3) に対して
a/(a+2b+3) + b/(b+2c+3) + c/(c+2a+3) ≧ G/(1+G).

JBMO2020って何だよ?

433 :132人目の素数さん:2020/05/27(水) 06:28:31.08 ID:2I72JytV.net
>>432
{a(a+2b+3)+ b(b+2c+3)+ c(c+2a+3)}= (a+b+c)(a+b+c+3)= 9A(A+1),
を左辺に掛ける。 コーシーにより
9A(1+A)(左辺)≧(a+b+c)^2 = 9AA,
∴ (左辺)≧ A/(1+A)≧ G/(1+G),   (← AM-GM)

JBMO = Junior Balkan Mathematical Olympiad

なお、日本のは JJMO

434 :132人目の素数さん:2020/05/27(水) 15:36:19 ID:2I72JytV.net
>>432
1/(1+x)≧ 1-x より
a/(a+2b+3) = a/{(a+b+c+3) + (b-c)}
 = a/{(a+b+c+3) (1+x)}
 ≧ a(1-x)/(a+b+c+3)
 = a/(a+b+c+3) - (ab-ca)/(a+b+c+3)^2,
循環的にたす。
 (左辺)≧(a+b+c)/(a+b+c+3)≧ G/(G+1)),

435 :132人目の素数さん:2020/05/27(水) 16:26:37.09 ID:2I72JytV.net
>>270 のことでござるか・・・・
 JBMO2020 はバルカンMO (BMO) のジュニア版のこと >>433
じゃなく、ただのハンドル名かも・・・・

436 :132人目の素数さん:2020/05/27(水) 18:38:24.11 ID:2I72JytV.net
〔問題3〕改
a,b,c は正の実数で 1/a + 1/b + 1/c ≧ 3 であるとする。
 (a+1/b)^2 + (b+1/c)^2 + (c+1/a)^2 ≧ 3(a+b+c+1).
を証明せよ。 等号が成立つのはいつか?

Problem 3. (modified)
 Let a,b,c be positive real numbers such that 1/a+1/b+1/c ≧3.
Prove that
 (a+1/b)^2 + (b+1/c)^2 + (c+1/a)^2 ≧ 3(a+b+c+1).
When does equality hold ?
 JBMO-2014 P.3

437 :132人目の素数さん:2020/05/27(水) 19:07:41.72 ID:2I72JytV.net
相加平均 A =(a+b+c)/3, 
調和平均 H = 3/(1/a+1/b+1/c)とおく。
問題の条件は H≦1.
2乗平均≧相加平均 より
 (左辺)≧ 3(A + 1/H)^2
 = 3(A-1)^2 + 3(3A+1) + 3(A-H)(2-H)/H + 3(1/H^2 - H)
 ≧ 3(3A+1)
 = 3(a+b+c+1),

http://global.olympiadsuccess.com/junior-balkan-mathematical-olympiad
  → JBMO 2014 Solutions (マケドニア大会)

438 :132人目の素数さん:2020/05/30(土) 19:23:31 ID:vR2Jo4eU.net
[例9-3] 改
 次の不等式をみたす整数a,b,cで、どれか1つは0でなく、
かつどの絶対値も100万を超えないものが存在することを示せ。
  |a + b√2 + c√3|< 10^(-12),

[第2章.274-276]
秋山 仁 + ピーター・フランクル 共著:
[完全攻略]数学オリンピック, p.47-48, 日本評論社 (1991/Nov)

注)鳩ノ巣原理では解けません。

439 :132人目の素数さん:2020/05/31(日) 08:48:01.74 ID:vwaCsVj1.net
二変数a,bの相加平均、相乗平均、調和平均の幾何学的な証明(直径a+bの円)がありますけど
三変数a,b,cの場合の幾何学的な証明はあるのでしょうか?相加平均は重心という自明な意味が
あるけど残りの二つの幾何的な意味がよくわからない。

440 :132人目の素数さん:2020/05/31(日) 11:46:33.00 ID:keH19MaE.net
自然に考えたら球に拡張しそうだけど

441 :132人目の素数さん:2020/05/31(日) 16:56:40.83 ID:YoOSwytJ.net
平面でやりたいのでは?

442 :132人目の素数さん:2020/05/31(日) 17:06:28.95 ID:vwaCsVj1.net
球だとできるんですか?立方体で無理やり解釈することもできるけど一目瞭然では全然ないようなのしかできない。。

3辺がa,b,cの直方体の体積V=abc ,表面積S=2(ab+bc+ca) とすると
3辺の長さの相加平均≧直方体と同体積の立方体の1辺≧V/表面積の平均値

443 :132人目の素数さん:2020/06/01(月) 03:27:29.95 ID:LHxMDESI.net
>>438
97 -56√3 = 1/(97+56√3) = 0.005154776
99 -70√2 = 1/(99+70√2) = 0.005050634
辺々足して14で割る。
14 - 5√2 - 4√3 = 7.28957859×10^(-4) ・・・・ (1)
辺々引いて2で割る。
-1 + 35√2 - 28√3 = 5.207113×10^(-5) ・・・・ (2)
(2)×14 - (1)
-28 + 495√2 - 388√3 = 3.7957659×10^(-8) ・・・・ (3)
また、
127 + 138√2 -186√3 = 2.139967594×10^(-5) ・・・・ (4)
205 - 58√2 - 71√3 = 6.04497020×10^(-6) ・・・・ (5)

* 3.352882344113・・・・×10^(-13)まではあるらしい。

444 :132人目の素数さん:2020/06/02(火) 03:49:20.04 ID:TPydHgX/.net
>>438
a=96051, b=-616920, c=448258 のとき
 a + b√2 + c√3 = 3.352882344113・・・×10^(-13)

445 :132人目の素数さん:2020/06/02(火) 03:56:28 ID:TPydHgX/.net
〔問題404-627〕
0<x<y<1<x+y のとき
 {(1-x)(1-y)(x+y-1)(y-x)^2}/(x+y)^2
の最大値を求めるにはどうすればいいでしょう?

[高校数学の質問スレPart404.627,632,635]

446 :132人目の素数さん:2020/06/02(火) 04:18:56.72 ID:TPydHgX/.net
〔問題404-634〕
q>1 を定数とする。
x^q + y^q + z^q = 1 を満たす正の実数x,y,zであって
(x+1)(y+1)(z+1) を最大にするものを求めよ。

[高校数学の質問スレPart404.634,639,648]

447 :132人目の素数さん:2020/06/02(火) 05:10:30.68 ID:Zifnd/u1.net
log(1+t^(1/q))はこのスレでは流行らないんだよな。

448 :132人目の素数さん:2020/06/02(火) 06:27:38 ID:fgXdb1VU.net
ツイッターで拾ったこの問題の最後の問題5

https://i.imgur.com/LgaZhH2.png

449 :132人目の素数さん:2020/06/02(火) 12:22:08 ID:TPydHgX/.net
問題1
 三角形ABCとその内部の点Pは、AB=7、AC=8、PB=1、PC=4 を満たす。
∠BAC と ∠BPC の二等分線が平行であるときの、BCの長さを求めよ。

(解答例)
題意より、点Pは△ABCの垂心となる。
 AP⊥BC、BP⊥CA、CP⊥AB
二等分線の方向をx軸とすれば傾きは
 AP -1/5、 BC 5
 BP -3、 CA 1/3
 CP 3、 AB -1/3
よって
 A(0,0) B(21/√10, -7/√10) C(24/√10, 8/√10) P(20/√10, -4/√10)
長さは
 BC = 3√(13/5) = 4.83735
 AP = 4√(13/5) = 6.4498

450 :132人目の素数さん:2020/06/02(火) 14:08:28.88 ID:TPydHgX/.net
問題2
 nを4以上の整数とする。
ある正n角形の各頂点にはある頂点から反時計回りで1からnの整数が
書かれている。
この正n角形にn-3本の対角線(辺は含まない)を どの二つの対角線も
交わらないように取ると、正n角形はn-2個の三角形に分けられる。
(証明不要)
これらn-2個の三角形それぞれの得点をその三角形の3頂点に書かれた
整数の和とする。
n-2個の三角形の得点の総和として考えられる最大の値を求めよ。

(解答例)
Σ[k=1,n] (点 'k' を共有する三角形の数)
= Σ[k=1,n] (点 'k' を端点とする対角線の数+1)
∴ 点 'n' を端点とする対角線n-3本をとれば 2n(n-2)  (最大)

逆に 点 '1' を端点とする対角線n-3本を取れば (n+3)(n-2) (最小)

451 :132人目の素数さん:2020/06/02(火) 14:27:03.25 ID:TPydHgX/.net
問題3
 以下の等式を満たす正の整数の組 (a,b,c) を全て求めよ。
   a^(bc) + b^(ca) = c^(ab)

(解答例)
 a^(bc) < c^(ab) より a^c < c^a,
 b^(ac) < c^(ab) より b^c < c^b, 
∴ a^(1/a), b^(1/b) < c^(1/c),
一方
 1 < n^(1/n) < ・・・・ < 5^(1/5) < 4^(1/4) = 2^(1/2) < 3^(1/3),
と比べて
(a,b,c) = (1,1,2) (1,2,3) (2,1,3)

452 :132人目の素数さん:2020/06/02(火) 17:47:39.85 ID:TPydHgX/.net
問題4
 内接円を持つ四角形ABCDの辺 AB, BC, CD, DA 上に
それぞれ P, Q, R, S をとり、線分PRとQSの交点をKとする。
四角形 APKS, BQKP, CRKQ に内接円が存在するとき、
四角形 DSKR にも内接円が存在することを示せ。

(解答例)
 内接円をもつ ⇔ 2組の対辺の和が等しい。
だけでは解けぬ。どうするか?

453 :132人目の素数さん:2020/06/02(火) 18:21:12 ID:TPydHgX/.net
問題5
 ある2以上の整数dは、ちょうどk個の正の約数 d_1 < d_2 < ・・・・・ < d_k を持ち、
任意のk個の正の実数 x_1, x_2, ・・・・・, x_k に対して以下の不等式を満たす。
このようなdを全て求めよ。

(√x_1 + √x_2 + ・・・・・ + √x_k + 1)^(2d)
  ≦ (k+1)^(2d)・Π[i=1,k] {(x_i^d_i + k)/(k+1)}^d_(k+1-i)
  ≦ 2^(2d-k)・Π[i=1,k] (x_i^d + k^d_(k+1-i)),

ガラパゴス不等式と名付けたい・・・・

454 :132人目の素数さん:2020/06/03(水) 14:14:42 ID:Vj2o+qIA.net
[第7章.946]
 i-j=k をみたすn-k項と i-j=n-k をみたすk項 の計n項で Jensen する。
 n項の引数の和はkとなる。
Σ[i-j=k] f(x_i - x_j) + Σ[i-j=n-k] f(1 - x_i + x_j) ≧ n f(k/n),
k=1,2,・・・・,n-1 でたす。

--------------------------------------------------------
蛇足だが・・・・
 f(a) ≦ n ∫[a-1/2n, a+1/2n] f(x) dx,
より
 Σ[k=1,n-1] f(k/n) ≦ n∫[1/2n, 1-1/2n] f(x) dx ≦ (n-1)∫[0,1] f(x) dx,

http://suseum.jp/gq/question/2724

455 :132人目の素数さん:2020/06/04(木) 03:52:52.71 ID:UZmO2K4a.net
>>445
s = x+y, t=xy とおくと
 0 < t < 1 < s < 2,
16(1-x)(1-y)(x-y)^2 = 16(1-s+t)(ss-4t)  (← tの2次式)
 = (2-s)^4 - {(2+s)^2 -8t -8}^2   (← 平方完成)
 ≦ (2-s)^4,
より
(与式)≦ (s-1)(2-s)^4 /(16ss)
 = (2/√3 -1)^3 - g(s)(s-2/√3)^2 /(16ss)
 ≦ (2/√3 -1)^3
 = 0.003702332976 = M,

等号は s = 2/√3 = 1.1547 のとき。

10+M = 10 + (2/√3 -1)^3
 = (2/√3 +1)^3
 = 1/{3(2/√3 -1)}^3
 = 1/(27M),

M = 1/{27(10+M)} < 1/270 = 0.00370370・・・・

g(s) = {(√3)(2-s)^3 + (3√3 -4)(2-s)^2 + 4(3√3 -5)(2-s) + 8(7-4√3)}/(√3)
 > 8(7-4√3)/√3
 = 0.331615  (s<2)
∵ 5/3 < √3 < 7/4,

456 :132人目の素数さん:2020/06/04(木) 04:14:53.02 ID:UZmO2K4a.net
>>446
 q乗平均Q ≧ 相加平均A
より
 (x+1)(y+1) = (A+1)^2 - (1/4)(x-y)^2
  ≦ (A+1)^2
  ≦ (Q+1)^2,
∴ (x+1)(y+1)(z+1) ≦ (Q+1)(Q+1)(z+1),
∴ もし最大値があるとすれば、それは x=y=z に限る。

457 :132人目の素数さん:2020/06/04(木) 17:35:49 ID:UZmO2K4a.net
>449
△ABCの垂心をHとおく。
辺BCに関してHと対称な点をDとおくと、
∠D = ∠BHC = 180゚ - ∠A,
∴ Dは△ABCの外接円上にある。
 OA = OD
∴ ∠OAH = ∠ODH  ・・・・ (1)

△ABCを中点三角形とするような大三角形△A'B'C'を考える。
その垂心は三角形ABCの外心Oである。
相似関係より、
 ∠BAO=∠CAH, ∠CAH=∠BAO,
∴ ∠A の二等分線は ∠OAHの二等分線。
 ∠BDO=∠CDH, ∠CDH=∠BDO,
∴ ∠D の二等分線は ∠ODHの二等分線。
(1) より
 ∠BHC の二等分線 // ∠Aの二等分線

458 :132人目の素数さん:2020/06/06(土) 04:26:40 ID:klZxi4yn.net
[AMM, Problem 12154]
Let r_a , r_b , and r_c be the exradii of a triangle with circumradius R and inradius r. Prove
 r_a/(r_b + r_c) + r_b/(r_c + r_a) + r_c/(r_a + r_b) ≧ 2 - r/R.

459 :132人目の素数さん:2020/06/08(月) 02:55:50.02 ID:4nsS10XA.net
>>443
38419 -13895√2 -10836√3 = 9.489944×10^(-9),
1920 -42258√2 +33395√3 = 4.066451×10^(-10),

460 :132人目の素数さん:2020/06/08(月) 04:37:40.73 ID:4nsS10XA.net
>>443
97-56√3 = (2-√3)^4 = 1/(2+√3)^4,
99-70√2 = (√2 -1)^6 = 1/(1+√2)^6,
より
-28 +495√2 -388√3 = {-(√2 -1)^12 +(2-√3)^8}/28, ・・・・ (3)

461 :132人目の素数さん:2020/06/09(火) 10:39:14.84 ID:oCR5MqlE.net
38419 -13895√2 -10836√3 = 9.489944×10^(-9)  ・・・・ (6)
1920 -42258√2 +33395√3 = 4.066451×10^(-10)  ・・・・ (7)

(4)×2 - (5)×7
 -1181 +682√2 +125√3 = 4.84560485×10^(-7)  ・・・・ (8)

(6)×4 - (3)
 153704 - 56075√2 -42956√3 = 2.11768032×10^(-12)  ・・・・ (9)

462 :132人目の素数さん:2020/06/15(月) 09:49:34.02 ID:m4MzqaBi.net
>>377
 (1,n) から 1 ≧ y1
 (i,i) から 2-S_i ≧ yi,
 (n,n) から 1 ≧ yn,
 (i,j) i<j から
  S_(i-1) ≧ yj または 2-S_j ≧ yi  ・・・・ これが難解

なお (1,2) 〜 (1,n-1) と (i,j) i>j は不要

 S_k = x0 + x1 + x2 + ・・・・・ + xk,

463 :132人目の素数さん:2020/06/28(日) 15:44:40 ID:OH7XqlAJ.net
(1) 円周率πに対して、3.1<π<3.2を示せ
(2)ネイピア数eに対して、2.7<e<2.8を示せ

464 :132人目の素数さん:2020/06/29(月) 17:05:05 ID:4ejNywyM.net
(1)
>>102 で θ=π/6 とおくと
 18/(2 + 2 + √3) < π < 2(1/2 + 1/2 + 1/√3),
 3.140237343 < π < 3.15470054

(2)
特に x=1 のとき、剰余項を入れて書けば
  e = 1 + 1/1! + 1/2! + ・・・・ + 1/n! + R_(n+1)   (11)
   R_(n+1) = e^θ/(n+1)! < 3/(n+1)!
今(11)を用いて1/n!を計算して行けば、n=4 までは右図のようになる。
それらを加えてeの近似値を得るが、剰余項 R_5 < 1/40 だから
 2 + 17/24 < e < 2 + 11/15
 2.7083333 < e < 2.7333333

高木:「解析概論」改訂第三版、岩波書店 (1961)
 第2章 微分法、§25.Taylorの公式、p.66

465 :132人目の素数さん:2020/06/29(月) 19:07:55.63 ID:4ejNywyM.net
(1)
Simpson の方法 は(3)の応用である。
h=(b-a)/2n と置いて y_(2i-1) に隣る二つの区間に関する積分∫f(x)dx の
近似値として(3)のように
   (h/3){y_(2i-2) + 4y_(2i-1) + y_(2i)}
を取って i=1,2,・・・・,n に亘って総計すれば
 ∫[a,b] f(x)dx
  ≒ (h/3){y_0 +4y_1 +2y_2 +4y_3 + ・・・・ + 2y_(2n-2) + 4y_(2n-1) + y_(2n)},
これが Simpson の公式である。              ・・・・ (5)
 もしも(4)によって剰余項をも取るならば、総計して
 R = -{n(h^5)/90}f^(4)(ξ) = -{(b-a)(h^4)/180}f^(4)(ξ),
これは Simpsonの公式の誤差の限界を与える。
 一例として π/4 = ∫[0,1] 1/(1+x^2) dx からπの近似値を計算してみよう。
n=5 とすれば h=0.1
 π/4 = (0.1/3)(1/1.00 + 4/1.01 + 2/1.04 + 4/1.09 + 2/1.16 + 4/1.25
         + 2/1.36 + 4/1.49 + 2/1.64 + 4/1.81 + 1/2.00) + R
   = (0.1/3)・23.5619446 + R
   = 0.7853981535 + R
∴ π = 3.141592614 + 4R

 -1.333×10^(-5) < R < 1.667×10^(-6)
 {実際は R = 9.91264×10^(-9)}

高木:「解析概論」改訂第三版、岩波書店 (1961)
 第3章 積分法、§38.定積分の近似計算 p.127-128
 (貞治先生も筆の誤り?)

466 :132人目の素数さん:2020/07/02(木) 07:33:44 ID:VISrmZkI.net
u,v,w∈R^n
(||v||^2 ||w||^2 - (v,w)^2) ||u||^2 ≧ ||(w,u)v - (v,u)w||^2.

これは有名な不等式なん?

467 :132人目の素数さん:2020/07/02(木) 17:35:21.65 ID:ceNKIuAv.net
A_(i,j) = v_i w_j - v_j w_i (交代テンソル)とおくと
 成分は 1≦i<j≦n をわたります。
与式は
 ||A||^2 |u|^2 ≧ |(A・u)|^2 = (スカラー三重積)^2
で、コーシーの不等式です。
3次元の場合、右辺は(u,v,wが作る平行六面体の体積)^2 です。

・幾何学的解釈
 ∠(v,w) = a, ∠(w,u) = b, ∠(u,v) = c
とおくと与式は
 1 + 2cos(a)cos(b)cos(c) - cos(a)^2 - cos(b)^2 - cos(c)^2 ≧ 0,
すなわち
 4 sin((a+b+c)/2) sin((-a+b+c)/2) sin((a-b+c)/2) sin((a+b-c)/2) ≧ 0,
なので、角{a,b,c}の三角不等式です。

・参考
{u,v,w}がなす球面三角形の面積をSとすると
 {4 cos(a/2) cos(b/2) cos(c/2) sin(S/2)}^2
 = 4 sin((a+b+c)/2) sin((-a+b+c)/2) sin((a-b+c)/2) sin((a+b-c)/2)
 = 1 - cos(a)^2 - cos(b)^2 - cos(c)^2 + 2cos(a)cos(b)cos(c)
 = |1,cos(c),cos(b)|
  |cos(c),1,cos(a)|
  |cos(b),cos(a),1|
これはヘロンの公式の球面版と考えられます。(カニョリの式)

468 :132人目の素数さん:2020/07/02(木) 18:26:18.89 ID:ceNKIuAv.net
(訂正)
与式は
 ||A||^2 |u|^2 ≧ |(A・u)|^2
で、コーシーの不等式です。
3次元の場合、 ||A|| はv,wが作る平行4辺形の面積です。

・・・・・

>>467 の行列式の各行に |u|, |v|, |w|, 各列に |u|, |v|, |w| を掛けて元に戻せば
|(u, u) (u, v) (u, w)|
|(v, u) (v, v) (v, w)|
|(w, u) (w, v) (w, w)|
の形(Grammian)になり、
| u'|
|v'||u, v, w|
|w'|
= (スカラー三重積)^2
= (u,v,wが作る平行六面体の体積)^2
でした。

469 :132人目の素数さん:2020/07/03(金) 01:01:30.39 ID:nLp+q6d/.net
ありがとう。難しすぎて今は分からんけど。

470 :132人目の素数さん:2020/07/16(木) 22:03:57 ID:/5szD4rG.net
4(a^6 + b^6 + c^6) + 5(a^5b + b^5c + c^5a) ≧ (1/27)*(a+b+c)^6

471 :132人目の素数さん:2020/07/18(土) 02:28:47 ID:xiJ191gm.net
>>470
この手の同次数の不等式の係数というのは分母を払えば
(N個の係数1の単項式)≧(N個の係数1の単項式)
と書けるようになっていますよね?
このタイプ(特に巡回や対称の場合)の不等式に関して
どのようなとき不等式が成立するかの一般論ってあるんでしょうか

昔、考えたのは対称の場合で
指数のタイプ
(n,0,…,0)、(n-1,1,0,…,0)、…、(1,1,…,1)
に関してヤング図形的な半順序付けをしたとき
上の条件を満たすような係数になっていて、左辺の各単項に右辺の単項式への順序を下げる1対1対応があるとき不等式は成立するというものでした(これは確か証明できた)
これを使うと相加相乗などはすぐに示せます

逆に同次対称で上の係数条件を満たすような不等式はこの左右の順序を下げる1対1対応が存在するか?も考えたのですが確かこれには反例がありました

472 :132人目の素数さん:2020/07/18(土) 06:33:37.69 ID:/5BiIC+G.net
よくってよ

473 :132人目の素数さん:2020/07/18(土) 12:04:37.03 ID:dk77+tdw.net
2006年度東工大の問題

https://i.imgur.com/0aBFX3v.jpg

474 :132人目の素数さん:2020/07/19(日) 19:28:26 ID:WIR0lTu1.net
>>470
aa=A, bb=B, cc=C とおく。
(左辺) - (aa+bb+cc)^3
 ≧ 4(A^3 + B^3 + C^3) + 15ABC - (A+B+C)^3
 = 3{A(A-B)(A-C) + B(B-C)(B-A) + C(C-A)(C-B)}
 = 3F_1(A,B,C)
 ≧ 0,           (Schur-1)

∴ (左辺) ≧ (aa+bb+cc)^3 ≧ (1/27)(a+b+c)^6.

 (1+1+1)(aa+bb+cc) ≧ (a+b+c)^2  (コーシー) から。

475 :132人目の素数さん:2020/07/21(火) 17:57:10 ID:Q73vct+1.net
>>470
a^3 =A', b^3 =B', c^3 =C' とおく。
(左辺) = 4(a^6 + b^6 + c^6) + 5(a^5・b + b^5・c + c^5・a)
 ≧ (A'+B'+C'){(4/3)(A'+B'+C') +5abc}  (← 補題)
 = (A'+B'+C'){(1/3)(a+b+c)^3 + F_1(a,b,c)}
 ≧ (1/3)(A'+B'+C')(a+b+c)^3
 ≧ (1/3){(aa+bb+cc)(a+b+c)}^2,   (← コーシー)

〔補題〕
 a^5・b + b^5・c + c^5・a ≧ abc(a^3+b^3+c^3),
(略証)
 a^5・b + b^5・c + c^5・a
  ≧ (a^3+b^3+c^3)^2 /(a/b+b/c+c/a)  (← コーシー)
  = abc(a^3+b^3+c^3)^2 /(aac+bba+ccb)
  ≧ abc(a^3+b^3+c^3),      (← チェビシェフ)

あるいは
 (16a^5・b + b^5・c + 4c^5・a)/21 ≧ a^4・bc,  (← AM-GM)
巡回的にたす。                 (終)

476 :132人目の素数さん:2020/07/22(水) 07:47:23.22 ID:U4xy9LSi.net
〔演習問題1.90〕
a,b,c を非負実数とする。このとき、次を証明せよ。
 aa+bb+cc + 2abc + 1 ≧ aa+bb+cc + 3(abc)^(2/3)
  ≧ aa+bb+cc + 9abc/(a+b+c)
  ≧ 2(ab+bc+ca),

[9] 佐藤(訳), 朝倉書店 (2013) p.41

477 :132人目の素数さん:2020/07/22(水) 07:49:09.73 ID:U4xy9LSi.net
(略証)
左・中は AM-GM で出る。
右は通分して
 (a+b+c)(aa+bb+cc-2ab-2bc+2ca) + 9abc
 = a(a-b)(a-c) + b(b-c)(b-a) + c(c-a)(c-b)
 = F_1(a,b,c) ≧ 0,      (Schur-1)

478 :132人目の素数さん:2020/07/22(水) 14:59:40.78 ID:cBVWOvpo.net
a,b,c,d>0
a + (ab)^(1/2) + (abc)^(1/3) ≦ (4/3)*(a+b+c)
a + (ab)^(1/2) + (abc)^(1/3) + (abcd)^(1/4) ≦ (143/100)*(a+b+c+d)

Slovenia 2012 ( ゚∀゚) ウヒョッ!

479 :132人目の素数さん:2020/07/23(木) 14:32:02 ID:cdENLWJx.net
右辺の係数を λ_n とおくと
 λ_1 = 1.0
 λ_2 =(1+√2)/2 = 1.20710678118655 (a = (1+√2)^2・b)
 λ_3 = 4/3 = 1.333333333  (a = 4b = 16c)
 λ_4 = 1.42084438540961  (a = bp = cq = dr)
 ・・・・
[第8章.972-990]

[前スレ.041] あたりの【Kiran Kedlaya】はこれの改良版(?)

480 :132人目の素数さん:2020/07/23(木) 15:03:09.65 ID:cdENLWJx.net
>>475
〔補題〕
 a^{n+1}・b + b^{n+1}・c + c^{n+1}・a ≧ abc (a^{n-1} + b^{n-1} + c^{n-1}),

(略証)
 n=0 のとき等号成立
 n=1 のとき AM-GM
 n≧2 のとき
 a^{n-3} /b + b^{n-3} /c + c^{n-3} /a
 = (a^{n-2}・c + b^{n-2}・a + c^{n-2}・b) /(abc)
 ≦ (a^{n-1} + b^{n-1} + c^{n-1}) /(abc),
よって
 (左辺) ≧ (a^{n-1}+b^{n-1}+c^{n-1})^2 /(a^{n-3}/b + b^{n-3}/c + c^{n-3}/a)
                  (← コーシー)
 ≧ abc(a^{n-1}+b^{n-1}+c^{n-1}).

(別法)
 AM-GMで
 [nn・a^{n+1}・b + b^{n+1}・c + n c^{n+1}・a] /(nn+n+1) ≧ a^n・bc,
巡回的にたす。

481 :132人目の素数さん:2020/07/23(木) 20:57:22.44 ID:sHXJwhgv.net
異なる4つの実数が任意に与えられたとき
そこから |ab+1|>|a-b| を満たす異なる実数a,bが選べることを示せ。

482 :132人目の素数さん:2020/07/24(金) 22:44:47.85 ID:+6BjH0O/.net
>>481
tanで置き換えれば瞬殺

483 :132人目の素数さん:2020/07/25(土) 10:36:55.68 ID:g3fpMEvS.net
>>481

[高校数学の質問スレPart405] から。

4つの実数をx_iとする。(i=1〜4)
θ_i = arctan(x_i) (-π/2 < θ_i < π/2) とおく。
4つから上手く2つを選ぶと
 |θi - θj| ≦π/4 または |θi-θj±π| ≦π/4,
となる。
tanの加法公式
 tan(α-β) = (tanα-tanβ)/(1+tanα・tanβ),
より
 1 ≧ |tan(θi - θj)| = |(x_i-x_j)/(1+x_i・x_j)|,
よって x_i, x_j は条件を満たす。
等号成立は {x_i} がπ/4ずつ異なるとき。 [895,948-949]

例:tan(±22.5゚) = ±(√2 -1), tan(±67.5゚) = ±(√2 +1),

484 :132人目の素数さん:2020/08/05(水) 03:24:43 ID:ud0wEpwG.net
三角形ABCの外接円の半径Rと内接円の半径rに対して、
2/R ≦ (1/a)*sec(A/2) + (1/b)*sec(B/2) + (1/c)*sec(C/2) ≦ 1/r.

[AMM, Prob12168] ( ゚∀゚)ウヒョッ!

485 :132人目の素数さん:2020/08/10(月) 14:07:49 ID:WaC83EyQ.net
不等式botより

https://i.imgur.com/7iHjp4R.png

486 :132人目の素数さん:2020/08/11(火) 16:49:54.71 ID:sLooAqcf.net
〔Inequalitybot 98〕
aa ≦ 1, aa+bb ≦ 5, aa+bb+cc ≦ 14, aa+bb+cc+dd ≦ 30 のとき
   a+b+c+d ≦ 10
を示せ。   Hungary-Israel binational 2007, 1日目, 問2

487 :132人目の素数さん:2020/08/11(火) 17:00:55.87 ID:sLooAqcf.net
コーシーにより、
 (a+b+c+d)^2 /(1+2+3+4) ≦ aa + bb/2 + cc/3 + dd/4
 = (1-1/2)aa + (1/2-1/3)(aa+bb) + (1/3-1/4)(aa+bb+cc) + (1/4)(aa+bb+cc+dd)
 ≦ (1-1/2) + (1/2-1/3)・5 + (1/3-1/4)・14 + (1/4)・30
 = 10,
等号成立は (a,b,c,d) = (1,2,3,4)

488 :132人目の素数さん:2020/08/11(火) 22:11:26.18 ID:sSrJdUio.net
a,b,c∈R,
a^2 + b^2 + c^2 > 0,
-1/2 ≦ Σ[cyc] ab/(a^2 + b^2 + 3c^2) ≦ 3/5.

[不明] ( ゚∀゚)ウヒョッ!

489 :132人目の素数さん:2020/08/14(金) 08:44:28.46 ID:63pIFzc2.net
1824 アンケよろしく @gas1824s (2020/08/13 22:08:21)
回りくどいのでほかにいい解法ありませんかね #数学教えて
https://pbs.twimg.com/media/EfTYnxTUYAAefqC.jpg
http://twitter.com/gas1824s/status/1293897176909660161
(deleted an unsolicited ad)

490 :132人目の素数さん:2020/08/14(金) 09:01:51.38 ID:kEvbrh9S.net
(a^3+a^3+b^3)/3≧(a^3a^3b^3)^(1/3)=a^2b

491 :132人目の素数さん:2020/08/14(金) 09:49:28 ID:6FR6kR83.net
>>490
おい右辺w

492 :132人目の素数さん:2020/08/14(金) 14:32:03.90 ID:Vqud894y.net
>>489
a,b,c≧0 に対し
 a^3 + b^3 + c^3 ≧ abb + bcc + caa,
を示せ。
--------------------------------------
 差積 (a-b)(b-c)(c-a) の符号は正にも負にもなるから、
このままではマズイ。
(ついでに言えば、符号も変)

0 ≦ {(a+2b)(a-b)^2 + (b+2c)(b-c)^2 + (c+2a)(c-a)^2}/3
= a^3 + b^3 + c^3 -abb -bcc -caa,

とやるか又は AM-GM で

(a^3 + 2b^3)/3 - abb = (1/3)(a+2b)(a-b)^2 ≧ 0  >>490
を循環的にたす。

493 :132人目の素数さん:2020/08/18(火) 18:55:07 ID:Nf0TF7ck.net
そんなに難しくない問題

https://i.imgur.com/knbMWIu.jpg

494 :132人目の素数さん:2020/08/18(火) 21:30:28 ID:ymr8iYI5.net
「入試数学の純粋な難問」
0 ≦ x,y,z ≦ 1 のとき
 (x+y+z)/3 + √{x(1-x)+y(1-y)+z(1-z)} ≦ 3/2
を示せ。
--------------------------------------------------------
(x+y+z)/3 = A とおく。
x(1-x) = (3/2)(3/8 - x/3) - (x - 3/4)^2 ≦ (3/2)(3/8 - x/3),
より
x(1-x) + y(1-y) + z(1-z) ≦ (3/2)(9/8 - A)
 = (3/2 - A)^2 - (3/4 - A)^2 ≦ (3/2 - A)^2,
よって
√{x(1-x) + y(1-y) + z(1-z)} ≦ 3/2 - A,

495 :132人目の素数さん:2020/08/18(火) 22:13:30.32 ID:Nf0TF7ck.net
(x+y+z)/3=tとおいたら相加相乗平均でいけない?

496 :132人目の素数さん:2020/08/18(火) 23:14:15 ID:YNr/CS7O.net
続けて。

497 :132人目の素数さん:2020/08/26(水) 23:23:01 ID:oXbdk8QE.net
a,b,c>0
$\frac{a^3+b^3}{ \sqrt{a^2-ab+b^2} } + \frac{b^3+c^3}{ \sqrt{b^2-bc+c^2} } + \frac{c^3+a^3}{ \sqrt{c^2-ca+a^2} } \geq 2(a^2+b^2+c^2)$

2020 China Norther MO ( ゚∀゚)ウヒョッ!

498 :132人目の素数さん:2020/09/01(火) 21:06:34.76 ID:2xmHjvQW.net
Twitterから色々拾ってきた
https://i.imgur.com/wm73ysp.png
https://i.imgur.com/WZ2vouq.jpg
https://i.imgur.com/mRGlGoa.jpg

499 :132人目の素数さん:2020/09/01(火) 22:58:10.89 ID:g1e3GOfL.net
うむ、よく訓練された不等式ヲタだな。

500 :132人目の素数さん:2020/09/02(水) 00:03:43.87 ID:3yJQ3R53.net
(aa+1)(bb+1)(cc+1) = (a+b+c-abc)^2 + (ab+bc+ca-1)^2.

501 :132人目の素数さん:2020/09/03(木) 00:51:04.79 ID:PGJ1gE8Y.net
(a+i)(b+i)(c+i) = (abc -a-b-c) + (ab+bc+ca-1)i,
(a-i)(b-i)(c-i) = (abc -a-b-c) - (ab+bc+ca-1)i,
辺々掛ける。

502 :132人目の素数さん:2020/09/03(木) 01:07:54.34 ID:PGJ1gE8Y.net
>>497
a,b,c>0 のとき
 (a^3+b^3)/√(aa-ab+bb) + (b^3+c^3)/√(bb-bc+cc) + (c^3+a^3)/√(cc-ca+aa) ≧ 2(a^2 + b^2 + c^2),

(略証)
コーシーで
 (x^3+y^3)/√(xx-xy+yy) = √{(x^3+y^3)(x+y)} ≧ x^2 + y^2,
巡回的にたす。

503 :132人目の素数さん:2020/09/03(木) 02:47:25.14 ID:yitbZe7d.net
>>501
からくりを見ると、当たり前の等式だったんだなあ ( ゚∀゚)ハァハァ…

504 :132人目の素数さん:2020/09/03(木) 13:48:56.02 ID:PGJ1gE8Y.net
>>500
実数でやるなら
 a+b+c = s, ab+bc+ca = t, abc = u,
とおく。
 (左辺) = (abc)^2 + ((ab)^2 + (bc)^2 + (ca)^2) + (a^2 + b^2 + c^2) + 1
 = uu + (tt-2su) + (ss-2t) + 1
 = (uu -2su +ss) + (tt -2t +1)
 = (u-s)^2 + (t-1)^2,

505 :132人目の素数さん:2020/09/03(木) 14:48:46.83 ID:PGJ1gE8Y.net
>>498
(上)
〔問題214〕
自然数n∈Nを固定する。
i=1,2,・・・・・,2n に対して |x_i| ≦ 1 の値をとるとき
 Σ[1≦r<s≦2n] (s-r-n) x_r x_s
の取り得る最大の値を求めよ。
 IMO Shortlist 2015 A-3
 Inequalitybot [214]


(中)
△ABCにおいて、
 F = {(sinA)^2 + 2(sinB)^2 + 3(sinC)^2} / {(sinA)(sinB)(sinC)}
とおく。
(1) △ABCの3辺の長さを BC=a, CA=b, AB=c とおき、
さらに△ABCの面積をSとする。
F を a,b,c,S で表わせ。
(2) Fの最小値を求めよ。

(下)
Problem 26
正の実数 a,b,c が ab+bc+ca=3 をみたすとき、
 a(bb+cc)/(aa+bc) + b(cc+aa)/(bb+ca) + c(aa+bb)/(cc+ab) ≧ 3
が成立することを示せ。

506 :132人目の素数さん:2020/09/03(木) 16:26:15 ID:PGJ1gE8Y.net
(中)
(1) 正弦定理
 sin(A) = a/2R, sin(B) = b/2R, sin(C) = c/2R,

 S = abc/4R,
より
 F = 2R(aa+2bb+3cc)/abc = (aa+2bb+3cc)/2S,

(2)
ところで 面積S は a,b,c の関数である。(ヘロンの公式)
 (aa+2bb+3cc)^2 - 11・16SS
 = (aa+2bb+3cc)^2 - 11{2(ab)^2 + 2(bc)^2 + 2(ca)^2 -a^4 -b^4 -c^4}
 = (3・4・5){(bb/4-cc/3)^2 + 2(cc/3-aa/5)^2 + 3(aa/5-bb/4)^2}
 ≧ 0,
 aa+2bb+3cc ≧ (4√11)S,
∴ F ≧ 2√11 = 6.63325
等号成立は a:b:c = √5:√4:√3 のとき。

507 :132人目の素数さん:2020/09/04(金) 01:17:36.55 ID:USkdw4WV/
【誰でもOK】手堅く月50万を稼ぐ手順
https://www.youtube.com/watch?v=QRRcJ3D-6uI
動画編集者をレベル分けしてみた
https://www.youtube.com/watch?v=Gq9-CmrpVHo
動画編集のディレクター需要が高まる3つの理由
https://www.youtube.com/watch?v=9qyECT9f_ZE
動画編集者のディレクターになるメリットとデメリット
https://www.youtube.com/watch?v=PxK-wqLGrWw
【体験談】動画編集のディレクターという職業
https://www.youtube.com/watch?v=PLshf0PJyNo
動画編集者がYouTubeをやる3つのメリット
https://www.youtube.com/watch?v=V_b_lfaEtwQ
動画編集初心者が勉強始めてから案件獲得するまでの3ステップ
https://www.youtube.com/watch?v=pEt7NGOqm4U
【動画編集】案件を得るための4つの営業先【超初心者向け】
https://www.youtube.com/watch?v=S_iyjOUq2ZI

508 :132人目の素数さん:2020/09/13(日) 23:58:13.39 ID:JpJgDqA9.net
a,b,c,d > 0
\sqrt[3]{ab} + \sqrt[3]{cd} ≦ \sqrt[3]{(a+c+d)(a+c+d)}

あばばばばばば
  ∩___∩         
  |ノ   ヽ/⌒)    あびゃば
  /⌒)(゚) (゚) /       あびゃあばばば
 / / (_●)ミ /         ∩――、
( ヽ |∪| /         /(゚)ヽ _ ヽ
 \  ヽノ /         / (● (゚) |つ
  /    /         | (入_ノ ミ
 |    /         | (_/  ノ
 | /\ \         \___ノ゙ー-、
 | /  ) )          /\    _ \
 (_ノ  ( \        (⌒O /\   (_ノ
     \_)        \ノ   /  、  )0

509 :132人目の素数さん:2020/09/14(月) 02:22:15.74 ID:8KYEGgmf.net
何かが変だ

510 :132人目の素数さん:2020/09/14(月) 04:14:30.54 ID:MMq0bu8b.net
何かおかしい、何となくそんな気がした。
TVに映る試合は俺とは全く縁もゆかりもない県同士の戦いだが、負けてる方をなんとな〜く応援している気分でいると、これまたなんとなくそろそろハルヒが騒ぎ出すような気がした。

511 :132人目の素数さん:2020/09/15(火) 07:33:01.17 ID:bL5lP9LW.net
>>488
等号成立条件だけ。
 最小値: {a,b,c} = {-1,0,1}
 最大値: {a,b,c} = {1,1,1} {1,1,2/3}

512 :132人目の素数さん:2020/09/15(火) 21:41:55.44 ID:oug42vb/.net
うむ

513 :132人目の素数さん:2020/09/15(火) 22:27:51.50 ID:5JiWovoE.net
うむ、エレガントな証明を見せてもらおうか。

514 :132人目の素数さん:2020/09/21(月) 22:00:12.84 ID:uwUcrYFn.net
>>488>>511
等号成立条件は、たぶんこうぢゃなゐかな? ( ゚∀゚)ウヒョッ!

(a,b,c,d) = (t, kt, (1+ 1/k)t, k(k+1)t), ただし k, t > 0 とする。

515 :132人目の素数さん:2020/09/22(火) 13:24:21.97 ID:7+NxYT1p.net
>>514
間違った。 >>513>>508 の等号成立条件。

516 :132人目の素数さん:2020/09/23(水) 14:21:02.58 ID:qMQmLmqf.net
xが0以上のとき 5x^3-3x+1>0

微分法で簡単に示せるですが
不等式エキスパートの人なら巧みな多項式変形とかで示せるですか?

517 :132人目の素数さん:2020/09/23(水) 21:07:53.46 ID:63e1O9oo.net
x√5 = X とおけば
 5x^3 - 3x + 1 > 5x^3 - 3x + (2/√5)
 = (X^3 - 3X + 2) /√5
 = (X+2)(X-1)^2 /√5
 ≧ 0,

ただし、x=1/√5 で極小になることを
微分などの方法で知る必要がある…

518 :132人目の素数さん:2020/09/23(水) 21:14:37.25 ID:yFt7+l2/.net
相加相乗平均だけで示せないかな

519 :132人目の素数さん:2020/09/23(水) 22:26:05.40 ID:UPCkCEIn.net
a≧b≧c≧d>0 かつ a+b+c+d=1 のとき、
(a+2b+3c+4d)(a^a)(b^b)(c^c)(d^d) ≦ 1.

520 :132人目の素数さん:2020/09/23(水) 22:35:07.36 ID:qMQmLmqf.net
x=0のときは明らかなのでx>0として 5x^2 + 1/x > 3 を言えばよいが
相加相乗で左辺≧3*(5/4)^(1/3) >3 。

式変形だけで、例えば
x^16 - x + 1 = (x^8-1/2)^2+(x^4-1/2)^2+(x^2-1/2)^2+(x-1/2)^2
みたいな感じの巧みな変形でいけないものでしょうか。

521 :132人目の素数さん:2020/09/24(木) 10:06:27.00 ID:qc+lGULo.net
>>518
 X^3 + 1 + 1 ≧ 3X,
は 相加相乗平均(AM-GM) と思ってもいいし、
コーシー
 (X^3 + 1 + 1)(1 + X^3 + 1)(1 + 1 + X^3)
 ≧ (X + X + X)^3
 = (3X)^3,
の3乗根と思ってもいい。

522 :132人目の素数さん:2020/09/24(木) 10:21:36.43 ID:3a+g1aMq.net
>>520-521
abc3数の相加相乗平均は、
(a+b+c)(a^2+b^2+c^2-ab-bc-ba)
= (a+b+c){(a-b)^2+(b-c)^2+(c-a)^2}
と変形できるので、a=X・b=1・c=1を代入して……って>>517と同じ式変形やないかーい笑

y=f(x) とした関数は非負だと x=1/√5 で極小値かつ最小値をとる。
極小値だけ移動した g(x)=f(x)-極小値 を考える→
x軸に接する
→(x-1/√5)^2 または (X-1)^2を因数にもつ
という考えと同じ。

523 :132人目の素数さん:2020/09/24(木) 22:14:11.72 ID:shPxNCvG.net
>>522
少し話が逸れるんだけど4次や5次の相加相乗にも同じように直接平方完成する変形があるんでしょうか?

524 ::2020/09/25(金) 01:49:19.72 ID:a6NySAb6.net
>>523
> 少し話が逸れるんだけど4次や5次の相加相乗にも同じように直接平方完成する変形があるんでしょうか?
なんと出来るらしい。
https://mathoverflow.net/questions/279969/wanted-positivity-certificate-for-the-am-gm-inequality-in-low-dimension

AMとGMの差は非負多項式で表すことが可能 https://gyazo.com/0e13cfb59b28c529dd5adbfed354bd16
具体的な恒等式(5次) https://gyazo.com/58b89593fc30a48f70ab35cee68d31e5
具体的な恒等式(2次・3次・4次)https://gyazo.com/4726111c57e1863fca1b9fcd64678b23

アドルフ・フルヴィッツによる1891年の論文
Hurwitz, A. (1891). Ueber den Vergleich des arithmetischen und des geometrischen Mittels. Journal für die reine und angewandte Mathematik, 108, 266-268. https://link.springer.com/chapter/10.1007/978-3-0348-4160-3_35
nが奇数の場合を示したらしい: 藤原和将・小澤徹(応用物理)による2014年の論文 Fujiwara, Kazumasa, and Tohru Ozawa. Identities for the Difference between the Arithmetic and Geometric Means, (2014).
http://m-hikari.com/ijma/ijma-2014/ijma-29-32-2014/ozawaIJMA29-32-2014.pdf
nが偶数の場合を示しているらしい: ハーディ・リトルウッド・ポリア『不等式』第2章

なおこの問題は、ヒルベルトの第17問題(いつも正の有理式は平方和で表せる)の特殊な場合でもある。

最初に見付けたのページは医師でアマチュア数学者の佐藤郁郎によるコラムだったが、いかんせん読みにくく参考までに。(NGなのでURL貼らない)
因数分解の算法(その11)
因数分解の算法(その14)
因数分解の算法(その18)

525 :132人目の素数さん:2020/09/25(金) 02:15:38.40 ID:Bm3x9keW.net
>>524
めっちゃ詳しくありがとうございます!
まさか本当に出来るとは驚きです
第17問題のwikiを読んだ感じでは非負な斉次多項式に対して一般にこういうことは出来ないようですね
この不可能性はモデル理論的な話があるようでこれも面白そうです

526 :132人目の素数さん:2020/09/25(金) 06:52:57.18 ID:kUKUaPs5.net
>>524
ウホッ!興奮してきた…

527 :132人目の素数さん:2020/09/25(金) 13:45:47.22 ID:C/C9yJEj.net
(x_1)^n, ・・・・・, (x_n)^n の相加平均をA, 相乗平均をG,
兩n = n(A^n - G^n) = Σ x^n - nΠ x,
とおく。
兩2(a,b) = aa +bb -2ab = (a-b)^2,

兩3(a,b,c) = a^3 +b^3 +c^3 - 3abc
 = (a+b+c){(a-b)^2 + (b-c)^2 + (c-a)^2}/2,

兩4(a,b,c,d) = a^4 +b^4 +c^4 +d^4 - 4abcd
 = (aa-bb)^2 + (cc-dd)^2 + 2(ab-cd)^2
 = (aa-cc)^2 + (bb-dd)^2 + 2(ac-bd)^2
 = (aa-dd)^2 + (bb-cc)^2 + 2(ad-bc)^2,

兩5(a,b,c,d,e)
 = a^5 + b^5 + c^5 + d^5 + e^5 -5abcde
 = (a-b)(a^4 -b^4)/4 + (a-c)(a^4 -c^4)/4 +
 ・・・・・ + (d-e)(d^4 -e^4)/4
 + a兩4(b,c,d,e)/4
 + b兩4(c,d,e,a)/4
 + c兩4(d,e,a,b)/4
 + d兩4(e,a,b,c)/4
 + e兩4(a,b,c,d)/4,

* (x-y)(x^4-y^4) = (x+y)(xx+yy)(x-y)^2 ≧ 0,

528 :132人目の素数さん:2020/09/26(土) 22:14:27.50 ID:lnmePYpg.net
>>527
なるほど、5次の場合は
1/4(x+y)(x^2+y^2)(x-y)^2
=1/6(x^3+y^3)(x-y)^2+1/12(x+y)(x^2-y^2)^2
を利用すると>>524の藤原小澤の表示と一致するのか
藤原小澤の論文は流し読みしたけどテクすぎて全然わからん
表示の仕方の自由度高そうだし何か行列式的な表示とか対称式の空間上の作用素みたいなのを見つけて綺麗に示せないもんかね

>>525
訂正
てっきり不可能性がモデル理論的に分かると思っていたけど今日モデル理論の本借りて見てみたら肯定的な証明が書かれていた
17問題が肯定的なのか否定的なのか混乱してきた…

529 :132人目の素数さん:2020/09/26(土) 22:49:49.44 ID:yIQAC3t7.net
元の十七問題はΣ多項式^2でかけるか?でそれは誰かの反例が出た
後に実閉体まで話広げるとΣ実閉包の元^2でかける事が証明された(Artin)
永田先生の可換体論の5章に証明がある

530 :132人目の素数さん:2020/09/26(土) 23:07:14.49 ID:lnmePYpg.net
自分の読んでる「幾何学的モデル理論入門」(最近、改訂版が出たばかりらしい)に実閉体の第17問題が肯定的に解けることを利用して有理数体の場合も証明できるかのように書いてるように見えるんです…

531 :132人目の素数さん:2020/09/26(土) 23:15:05.52 ID:yIQAC3t7.net
>>624
どんなステートメントが書いてあるんですか?

532 :132人目の素数さん:2020/09/26(土) 23:32:33.79 ID:lnmePYpg.net
こんな感じです
微妙な言い回しなんでもしかしたら肯定的に言ってるわけではないかも…
https://dotup.org/uploda/dotup.org2265448.jpg

533 ::2020/09/26(土) 23:44:50.71 ID:lzfEoiqI.net
>>528
> 17問題が肯定的なのか否定的なのか混乱してきた…
一部が肯定的に解決された(部分的に可能)
https://i.imgur.com/Jit5Abl.jpg

534 :132人目の素数さん:2020/09/26(土) 23:49:36.06 ID:yIQAC3t7.net
>>532
thx
確か上がってる反例はΣ整式^2では表せない例でΣ有理式^2では表せるんだったかな?
Artinの定理の正確なステートメント覚えてない(かつ永田先生の本が現在部屋のどこにあるかわからん)のでわかんないけどΣ有理式^2で表すのは有理係数でもいけるのかもしれない

 任意の実閉包の中で0以上 →Σ有理式^2で表示できる

だったかも
確か右が言えてない場合に標数0の加法的付値体て0未満になる構造が存在する事示してそれを上手いこと微調整して通常のRの中で0以下に出来る事を示すんだったような

535 :132人目の素数さん:2020/09/26(土) 23:56:47.06 ID:lnmePYpg.net
あー、有理式なら肯定的という可能性があるんですね
ありがとうございます

536 :132人目の素数さん:2020/09/27(日) 00:37:31.86 ID:KH9c7ePZ.net
3次のAM-GM差の有理式の平方和表現はどうなるんだろう

537 :132人目の素数さん:2020/09/27(日) 00:49:47.04 ID:KH9c7ePZ.net
あ、全ての有理数に対して非負な多項式で考えるから奇数次の場合は全ての変数を平方にしておかないといけなくて藤原小澤の結果の変数を平方にするだけか

538 :132人目の素数さん:2020/09/27(日) 01:09:34.88 ID:KH9c7ePZ.net
いや、そうなってくると藤原小澤も必要なくて古典的なフルヴィッツの形で示せてるのか
無知すぎてスマン

539 :132人目の素数さん:2020/09/27(日) 01:15:18.97 ID:T1BBTchP.net
有理式使っていいならAM-GMは簡単でしょ?
いわゆる2冪でやっといて減らす作戦でいける

540 :132人目の素数さん:2020/09/27(日) 14:36:59.49 ID:N42SrDUa.net
>>527
兩n(a_1, a_2, ・・・・, a_n)
 = (a_1)^2 + (a_2)^n + ・・・・ + (a_n)^n - n(a_1)(a_2)・・・・(a_n)
 = Σ[i<j] (a_i - a_j)[(a_i)^{n-1} - (a_j)^{n-1}] /(n-1)
 + Σ[i=1,n] a_i (Σ[j≠i] (a_j)^{n-1} - (n-1)Π[j≠i] a_j) /(n-1)
 = Σ[i<j] (a_i - a_j)[(a_i)^{n-1} - (a_j)^{n-1}] /(n-1)
 + Σ[i=1,n] a_i 兩{n-1}(i以外) /(n-1),

541 :132人目の素数さん:2020/09/27(日) 14:42:24.65 ID:hueUvkJf.net
ある人の作った問題

https://i.imgur.com/HLrsj0N.jpg

542 :132人目の素数さん:2020/09/27(日) 15:07:23.11 ID:Upg5ciqK.net
ある人って誰かね?

543 :132人目の素数さん:2020/09/27(日) 22:25:26.77 ID:KH9c7ePZ.net
>>541
n=2のときが本質で、他は帰納法でしょうか

544 :132人目の素数さん:2020/09/28(月) 01:14:12.03 ID:VdFe70Zi.net
>>541
数列 {a_n}n∈N と {b_n}n∈N が |a_n|≦1, |b_n|≦1 (∀n∈N) を満たす時、
次を示せ。
| Π[i=1,n] a_i - Π[k=1,n] b_k | ≦ Σ[j=1,n] |a_j - b_j| (∀n∈N)

(略証)
Π[i=1,n] a_i - Π[k=1,n] b_k
 = Σ[j=1,n] (Π[i=1,j-1] a_i) (a_j - b_j) (Π[k=j+1,n] b_k),
三角不等式により
(左辺) ≦ Σ[j=1,n] (Π[i=1,j-1] |a_i|) |a_j - b_j| (Π[k=j+1,n] |b_k|)
 ≦ Σ[j=1,n] |a_j - b_j|
 = (右辺),

545 :132人目の素数さん:2020/10/06(火) 20:34:14.40 ID:CqXEEU8P.net
〔問題944〕
a,b,c は相異なる正の数で、√a + √b + √c = 1 を満たす。
 f(x,y) = log(y/x) / (1/x - 1/y),
に対して、
 f(a,b) + f(b,c) + f(c,a) ≦ 1/3
を示せ。

高校数学の質問スレPart407 - 944

546 :132人目の素数さん:2020/10/12(月) 12:43:43.49 ID:xle55bMk.net
IMO2020ロシア大会第2問に不等式問題

https://i.imgur.com/5THoJvk.png

547 :132人目の素数さん:2020/10/13(火) 14:51:16.00 ID:Aceyovpj.net
>>545
0<x,y, x≠y のとき
f(x,y) = log(y/x)/(1/x - 1/y)
 = log(y/x)/(√(y/x) - √(x/y))
 = 2t/(e^t - e^{-t})・√(xy)
 = t/sinh(t)・√(xy)
 ≦ √(xy),
 等号成立は x=y のとき。

(左辺) = f(a,b) + f(b,c) + f(c,a)
 ≦ √(ab) + √(bc) + √(ca)
 ≦ (1/3)(a+b+c + 2√(ab) + 2√(bc) + 2√(ca))
 = (1/3)(√a + √b + √c)^2,
 等号成立は a=b=c のとき。

548 :132人目の素数さん:2020/10/13(火) 23:32:20.28 ID:nOU2oznJ.net
RMM、SP346のような抽象的なのは、どこから手を付けていいか分からんちんぽ。
http://www.ssmrmh.ro/wp-content/uploads/2020/10/24-RMM-SPRING-EDITION-2022-1.pdf

549 :132人目の素数さん:2020/10/13(火) 23:54:02.78 ID:w92dB+J+.net
>>548
うほっ大量の不等式!

550 :132人目の素数さん:2020/10/14(水) 04:12:15.55 ID:PHtzabu1.net
JP346.
 両辺に ab(a+b) >0 を掛けて通分すると
ab(a+b)(左辺 - 右辺) = (a-b)^2 {(a-b)^2 + (4 - k/4)ab},
 (4 - k/4) ≧ 0,
 k ≦ 16,

551 :132人目の素数さん:2020/10/14(水) 19:11:56.93 ID:wXRh3JRG.net
>>550
いや知りたいのは、SP346でござるよ

552 :132人目の素数さん:2020/10/14(水) 20:10:00.68 ID:PHtzabu1.net
JP347.
基本対称式を
 s = a+b+c, t = ab+bc+ca, u = abc,
とおく。
(a+b)(b+c)(c+a) = st-u を掛けて通分すると
2(a+b)^2・(a+c)^2 + 2(b+c)^2・(b+a)^2 + 2(c+a)^2・(c+b)^2
 = 2(ss-t)^2 + 8su
 = s^4 + (5/3)tt + s(s^3 -4st+9u) + (tt-3su)/3
 ≧ s^4 + (5/3)tt,
∴ (左辺) ≧ {s^4 + (5/3)tt}/st = (s^3)/t + 5t/(3s),

JP348.
 a/b=x, b/c=y, c/a=z とおくと
 x^3 + y^3 + z^3 ≧ 3xyz = 3,   (← AM-GM)
 (x^4+y^4+z^4)(1+1+1) ≧ (x^2+y^2+z^2)^2, (←コーシー)
辺々掛ける。

553 :132人目の素数さん:2020/10/14(水) 22:07:28.25 ID:OfAfCbWz.net
>>551
346はkについての一次式だから
与式⇔k≦4次式÷4次式
になる(割る時、除数の符号わそこまで難しくない)
分子も分母も(a-b)^2で割り切れる

554 :132人目の素数さん:2020/10/15(木) 17:59:01.64 ID:2j40wcqC.net
JP350.
 (a+b+c)^2 ≦ 3(aa+bb+cc),
x ≧ 1/√3 のとき
{1 + 3x(1-x)}^2 - (4-3xx) = 3(3xx-1)(x-1)^2 ≧ 0,
√(4-3xx) + 3x(x-1) ≦ 1,
x=a,b,c でたす。

JP351.
ABCが鈍角凾フときは
 Πcos(・) ≦ 0,
 (左辺) ≧ 0 ≧ (右辺),
で成立するから以下では、ABCは鋭角Δとする。
 sin(A) = x, sin(B) = y, sin(C) = z
とおくと
 xyz > 0,
一方、題意より
 xx+yy+zz = 1 - 2xyz,
 xyz ≦ 1/8,
左辺に加法公式
 sin(A)sin(B) = cos(A)cos(B) - cos(A+B) = xy + z,
を入れれば
 (左辺) - (右辺) = (xy+z)(yz+x)(zx+y) - 4xyz + 5(xyz)^2
 = ・・・・,
 がんばれ

JP352.
△不等式で
 |a+b+c| + |a-b| + |a-c| ≧ |(a+b+c) + (a-b) + (a-c)| = 3|a|,
巡回的にたすと
 3|a+b+c| + 2(|a-b|+|b-c|+|c-a|) ≧ 3(|a|+|b|+|c|),

555 :132人目の素数さん:2020/10/15(木) 19:59:49.04 ID:2j40wcqC.net
UP346
 (左辺) = 1 + (x-1)exp(arctan(x))√(1+xx),  ←可積分
∴ x=1

UP347.
上の式は
0 = |x|^2 /m + |y|^2 /n - |x+y|^2 /(m+n)
 = xx' /m + yy' /n - (x+y)(x+y)' /(m+n),
mn(m+n) を掛けて通分する。
0 = nnxx' + mmyy' - mn(xy'+x'y)
 = (nx - my) (nx - my)'
 = |nx - my|^2
∴ x = (m/n)y,
これを下の式に入れて
 y = 1+2i,

UP534.
 (1/n)Σ[k=1,n-1] (k/n)sin(kπ/n)
 → ∫[0,1] x・sin(πx) dx   (n→∞)
 = [ sin(πx)/π^2 - x・cos(πx)/π ](x=0,1)
 = 1/π,

(与式) ≒ (1 + π/n)^n
 = {(1+π/n)^(n/π)}^π
 → e^π,  (n→∞)

UP359.
Ω = ∫[0,π/2] θ・cosθ/[(sinθ)^3 + (cosθ)^3] dθ
 = (π/36){π√3 + log(97/8 + 7√3)}
  - (1/144){ψ'(5/12) - ψ'(11/12)}
 = 0.71907287245537291248414214
ここに
ψ'(x) = {log(Γ(x))}" = {Γ'(x)/Γ(x)}'
 = Σ[k=0,∞] 1/(x+k)^2  … トリガンマ函数

556 :132人目の素数さん:2020/10/16(金) 05:20:43.69 ID:QJC/WS82.net
JP351.
 cos(A) = x, cos(B) = y, cos(C) = z
とおくと、
に修正…

SP349.
 s = -log(sin(a)) >0,  c = -log(cos(a)) >0,
とおく。
(左辺) = (e^{-s})^{√(c/s)} + (e^{-c})^{√(s/c)}
 = 2e^{-√(cs)}
 ≦ 2e^{-log(2)/2}    (*)
 = 2e^{log(1/√2)}
 = √2,
(*) a が端(0,π/2) に近づくとき cs は急に大きくなる。

SP351.
 (1+t)^{1/3} + (1-t)^{1/3}
 = 2/[(1+t)^{2/3} - ((1+t)(1-t))^{1/3} + (1-t)^{2/3})
 ≦ 2,

UP358.
|∫[a,b] e^{ix}/x dx | ≦ ∫[a,b] 1/x dx = log(b/a),
両辺を2乗する。

UP360.
 x = y = 1/3,

557 :132人目の素数さん:2020/10/16(金) 10:43:02.64 ID:QJC/WS82.net
http://www.551horai.co.jp/

(*) a が端(0,π/2) に近づくとき -√cs は大きくなる。(→0)

558 :132人目の素数さん:2020/10/16(金) 19:37:27.70 ID:QJC/WS82.net
SP353.
与式を辺々引くと
 λ(x-y) = √(λλyy-1) - √(λλxx-1)
  = λλ(yy-xx)/{√(λλyy-1) + √(λλxx-1)},
もし x-y≠0 ならば
 1 = - λ(x+y)/{√(λλyy-1) + √(λλxx-1)}
 < 0, (矛盾)
∴ x = y = z = 2/(λ√3),

559 :132人目の素数さん:2020/10/16(金) 20:49:16.21 ID:QJC/WS82.net
SP353. (別法)
t≧1 で f(t) = t + √(tt-1) は単調増加
与式より
 f(λx) = f(λy) = f(λz),
 λx = λy = λz,
λ>0 より
 x = y = z,

560 :132人目の素数さん:2020/10/17(土) 13:08:39.01 ID:Ml1qOBSK.net
UP346.
 f(x) e^{arctan(x)} = ∫ (2xx) e^{arctan(x)} / √(1+xx) dx
とおく。
 f '(x) + f(x)/(1+xx) = 2xx/√(1+xx),
 f '(x)√(1+xx) + f(x)/√(1+xx) = 2xx,
ここで
 f(x) = g(x)√(1+xx),
とおくと
 (1+xx)g '(x) + (1+x)g(x) = 2xx,
g(x) が n次多項式とすると
 g(x) = ax^n + ・・・
(左辺)= (n+1)ax^{n+1} + …
∴ n=1,
 (左辺) = 2ax^2 + g(1)(1+x),
 a=1, g(1)=0,
 g(x) = x-1,
 f(x) = (x-1)√(1+xx),

561 :132人目の素数さん:2020/10/18(日) 13:13:01.21 ID:ZEBeZlNg.net
SP354.
 log(x^{xy}・y^{yz}・z^{zx})
 = log(x^{xy}) + log(y^{yz}) + log(z^{zx})
 = y・log(x^x) + z・log(y^y) + x・log(z^z)
 ≦ y・(x^x - 1) + z・(y^y - 1) + x・(z^z - 1)
 = (y・x^x + z・y^y + x・z^z) - (x+y+z),

*) e^t ≧ 1+t より log(u) ≦ u-1,

562 :132人目の素数さん:2020/10/18(日) 19:36:39.48 ID:ZEBeZlNg.net
SP358.
コーシーで
 {(y+1)+(z+1)+(x+1)} {(z+1)(x+1)+(y+1)}{x^3/[(y+1)(z+1)] + cyclic}
 ≧ (x+y+z)^3
 = s^3
よって
 (左辺) ≧ 4s^3 /(s+3)^2 + 3
 = s{(2s/(s+3))^2 + (s+3)/2s + (s+3)/2s - 1}
 = s(3-1)       (← AM-GM)
 = 2s
 ≧ (右辺),
等号は s=3, x=y=z=1 のとき。

563 :132人目の素数さん:2020/10/18(日) 21:26:39.70 ID:ZEBeZlNg.net
SP358.
コーシーで
 { …… } {(z+1)+(x+1) + (y+1)}{ …… }
 ≧ (x+y+z)^3

 ≧ s(3-1)       (← AM-GM)

JP360.
 tan(x)^2/{tan(x)^3+cot(x)} + cot(x)^2/{cot(x)^3+tan(x)}
  - 2/{tan(x)^2 +cot(x)^2}
 = Σ {tan(x) + cot(x) -2}/{tan(x)^2 + cot(x)^2}
 = X / (XX+4X+2)
 ≦ 1/(4+2√2),           (*)
ここに X = tan(x) + cot(x) -2 ≧ 0,
∴ 0 ≦ (左辺) - (右辺) ≦ 3/(4+2√2),

*) (XX+4X+2) - (4+2√2)X = (X-√2)^2 ≧ 0,
等号成立は X = √2, sin(2x) = 2 - √2,

564 :132人目の素数さん:2020/10/19(月) 19:49:07.20 ID:wNxsDbEN.net
Twitterで拾った問題

https://i.imgur.com/nrEQJtL.jpg

565 :132人目の素数さん:2020/10/20(火) 20:12:27.86 ID:J8I4fsGY.net
〔問題558〕
正の実数 x,y,z が xyz=1 を満たすとき、以下を示せ。
x/(1+y+z)^3 + y/(1+z+x)^3 + z/(1+x+y)^3 ≧ 1/9 ≧ 1/(√xy + √yz + √zx)^2,

566 :132人目の素数さん:2020/10/22(木) 23:53:39.82 ID:F90OyzGB.net
>>565
問題番号がついてるけど、どこかのスレ番号で紹介されていたのかな?

567 :132人目の素数さん:2020/10/22(木) 23:54:13.75 ID:F90OyzGB.net
ああスマン。このスレの558の画像からか。

568 :132人目の素数さん:2020/12/03(木) 06:23:28.55 ID:qlrP4DQI.net
a,b,c > 1/2 のときに、aa+bb+cc+ab+bc+ca-a-b-c ≧0 を証明したい。
左辺を平方完成して、残り物 ab+bc+ca-3/4 にAM-GMする以外にハァハァできそうな方法はないかな?

569 :132人目の素数さん:2020/12/03(木) 06:24:46.82 ID:qlrP4DQI.net
>>564
左辺の分母の1を(abc)^(1/3)に変えて同次にするんだろうと思うけど、そこで手が止まっている…

570 :132人目の素数さん:2020/12/06(日) 02:09:47.83 ID:KT/cOuDT.net
>>568
 (左辺) = {(a+b)^2 + (b+c)^2 + (c+a)^2}/2 - (a+b+c)
   = {(a+b)(a+b-1) + (b+c)(b+c-1) + (c+a)(c+a-1)}/2
  > 0,
とか
 (左辺) = {4(a+b+c)^2 + (a-b)^2 + (b-c)^2 + (c-a)^2}/6 - (a+b+c)
 ≧ (2/3)ss - s       (s=a+b+c)
 = (2/3)s(s - 3/2)
 > 0,
とか色々あるけど、単に
 a' = a - 1/2 > 0, b' = b - 1/2 > 0, c' = c - 1/2 > 0,
でいい希ガス…

571 :132人目の素数さん:2020/12/07(月) 03:29:15.95 ID:8o9fiWHc.net
>>570
おぉ〜ありがとう!

572 :132人目の素数さん:2020/12/14(月) 01:35:46.45 ID:gLda82Cm.net
a,b>0は定数とする. 0<s,t<1のとき
st/(as+bt)+(1-s)(1-t)/{a(1-s)+b(1-t)}の最大値を求めよ.

573 :132人目の素数さん:2020/12/18(金) 06:19:31.83 ID:DAoaiwdi.net
1/(a+b) - (与式) = ab(s-t)^2 /[(a+b)(as+bt){a(1-s)+b(1-t}] ≧ 0,
等号成立は s=t のとき。

574 :132人目の素数さん:2020/12/18(金) 13:22:00.83 ID:DAoaiwdi.net
コーシーで
 st/(as+bt) ≦ (at+bs)/(a+b)^2,
 (1-s)(1-t)/{a(1-s)+b(1-t)} ≦ {a(1-t)+b(1-s)}/(a+b)^2,
辺々たす。

575 :132人目の素数さん:2020/12/19(土) 21:22:13.58 ID:iw6DTiTj.net
単位円に内接する正n角形のn個の頂点からの距離の和が最小になる点とその最小値を求めよ。

576 :132人目の素数さん:2020/12/20(日) 20:55:16.29 ID:QYPKWpxY.net
頂点A_k の極座標を (1, 2kπ/n) 点Pの極座標を (r, θ) とおく。
第二余弦定理より
PA_k = √{1 - 2r・cos(2kπ/n - θ) + rr} ≧ 1 - r・cos(2kπ/n - θ)
 等号成立は r=0 のとき
また
 Σ[k=1,n] cos(2kπ/n - θ) = Σ[k=1,n] {sin((2k+1)π/n - θ) - sin((2k-1)π/n - θ)}/{2sin(π/n)}
 = {sin((2n+1)π/n - θ) - sin(π/n - θ)}/{2sin(π/n)}
 = 0,
∴ Σ[k=1,n] PA_k ≧ n,
 等号成立は P=O のとき。

・nが偶数のとき (n=2m)
三角不等式より
 PA_k + PA_{m+k} ≧ A_k A_{m+k} = 2,
 等号成立は P が線分 A_k A_{m+k} 上にあるとき。(← 円の直径)
∴  Σ[k=1,n] PA_k = Σ[k=1,m] (PA_k + PA_{m+k}) ≧ Σ[k=1,m] 2 = 2m = n,
 等号成立は P=O のとき。

577 :132人目の素数さん:2020/12/20(日) 23:01:24.01 ID:QYPKWpxY.net
ヴェクトルの内積を使えば
OA_k = 1 より

PA_k ≧ ↑PA_k・↑OA_k = (↑OA_k - ↑OP)・↑OA_k
  = 1 - ↑OP・↑OA_k,

∴ Σ[k=1,n] PA_k ≧ n - ↑OP・{Σ[k=1,n] ↑OA_k} = n,
等号成立は ↑OP = o.

578 :132人目の素数さん:2021/01/01(金) 08:32:42.45 ID:NURKUP5N.net
      ∧_∧
     ( ´Д` )  新年あけまして
     /     ヽ
     し、__X__,ノJ

      /´⌒⌒ヽ
    l⌒    ⌒l  おめでとうございます。
   ⊂ (   ) ⊃
      V ̄V

正の数 a,b,c に対して
(a^2021 -a^3 +3)(b^2021 -b^3 +3)(c^2021 -c^3 +3) > (a+b+c)e,
e = 2.71828… は自然対数の底
>>299

579 :132人目の素数さん:2021/01/02(土) 13:31:16.81 ID:2x2Frbzp.net
>>578
eが出てくるのか…

580 :132人目の素数さん:2021/01/03(日) 06:38:55.14 ID:N51mYuOL.net
eが出てきても eじゃない…

(左辺) ≧ 2.7199579587(a+b+c)
等号は a=b=c = 0.9968783547581 のとき。

581 :132人目の素数さん:2021/01/03(日) 16:12:00.62 ID:+lbXmv47.net
すべての自然数nについて
Σ_{k=1}^n (k^(1/2)-1)≧(n/2)*((n/2)^(1/2)-1)
が成り立つことをしめせ

582 :132人目の素数さん:2021/01/03(日) 20:22:26.62 ID:N51mYuOL.net
k≧2 について
 (k-1)^3 - k(k-3/2)^2 = (3/4)k - 1 > 0,  (AM-GM)
 (k-1)^{3/2} > (k - 3/2)√k = k^{3/2} - (3/2)√k,
 √k > (2/3)(k^{3/2} - (k-1)^{3/2}),
(左辺) = Σ_{k=2}^{n} (√k - 1)
  > (2/3)(n^{3/2} - 1) - (n-1)
  = (1/3)(2√n +1)(√n - 1)^2
  > (n/2)(√(n/2) - 1),

583 :132人目の素数さん:2021/01/03(日) 22:48:34.64 ID:N51mYuOL.net
積分を使えば簡単だが…
√k > ∫_{k-1}^{k} (√x)dx = (2/3)(k^{3/2} - (k-1)^{3/2}),

584 :132人目の素数さん:2021/01/04(月) 20:59:39.27 ID:OQ8TTvGy.net
最後は
 (左辺) > n((2/3)√n - 1)
  > (n/2)(√(n/2) + √(n/3) - 2)  (← 補題)
  ≧ (n/2)(√(n/2) - 1)     (n≧3)
 n=1, n=2 は明らか。

〔補題〕
 1/√2 + 1/√3 < 4/3.
(略証)
コーシーで
 (1/√2 + 1/√3)^2 ≦ (1+1)(1/2+1/3) = 5/3 < 16/9 = (4/3)^2,

585 :132人目の素数さん:2021/01/05(火) 10:53:44.67 ID:7+Me0drG.net
GM-AM で
 1/√2 + 1/√n < 3/4 + (1/4 + 1/n) = 1 + 1/n,

586 :132人目の素数さん:2021/01/06(水) 18:50:24.31 ID:ZXZ11nuc.net
n が被ってしまった。。。。m 等にすべきか。
>>583 から
 Σ_{k=2}^{n} √k > √2 + ∫_[2}^{n} (√x) dx
  = √2 + (2/3)(n√n - 2√2)
  > (2/3)n√n - 1/2,        (n≧2)

587 :132人目の素数さん:2021/01/06(水) 19:13:16.60 ID:YFXd0DZ2.net
JMO2020本選5

588 :132人目の素数さん:2021/01/07(木) 14:00:54.18 ID:PX1JcXUq.net
>>579

正の数 a, b, c に対して
(a^2021 -a^3 +3)(b^2021 -b^3 +3)(c^2021 -c^3 +3) > (3/ln3)(a^3 +b^3 +c^3),

589 :132人目の素数さん:2021/01/09(土) 17:51:38.85 ID:Gni2ACgE.net
ついでに…

正の数 a, b, c に対して
(a^2021 - a^3 +3) (b^2021 - b^3 +3) (c^2021 - c^3 +3) > (1/(3ln3)) (a+b+c)^3,

http://suseum.jp/gq/question/3221

590 :132人目の素数さん:2021/01/10(日) 02:37:24.49 ID:k4Y9uhcW.net
〔問題〕
A, B, and C are non-negative real numbers. Prove that
3(A^4 + B^4 + C^4) ≧ (A^2 + B^2 + C^2)(AB + BC + CA) + (A^2 - B^2)^2 + (B^2 - C^2)^2 + (C^2 - A^2)^2
≧ (A^2 + B^2 + C^2)^2,

BMO-2016 Azerbaijan (改)
[数学オリンピック31.018]

591 :132人目の素数さん:2021/01/22(金) 05:19:03.42 ID:n9I3J2ea.net
>>589
 x^2021 - x^3 + 3 ≧ K^{1/3} {x^3 /(x。)^2 + x。+ x。},
 K = 0.30406358311 > 0.30341307554・・・ = 1/(3ln3),
 等号成立は x。= 0.99703312297 のとき。
あとはコーシーで。

http://www.casphy.com/bbs/highmath/472060/379,381
http://suseum.jp/gq/question/3221

592 :132人目の素数さん:2021/01/27(水) 16:42:45.22 ID:knjIwEAx.net
〔問題〕
凾フ各辺の長さを a,b,c とするとき、外接円の半径Rは
 (1/3)√(aa+bb+cc) ≦ R
  ≦ (1/(6√3)){a(b+c)/(b+c-a) + b(c+a)/(c+a-b) + c(a+b)/(a+b-c)},

佐藤淳郎(訳)「美しい不等式の世界」朝倉書店 (2013)
 (左) Leibnizの不等式 (定理2.4.5) p.88-89
 (右) 演習問題 2.57(改) p.94

593 :132人目の素数さん:2021/02/13(土) 23:28:41.35 ID:DMjAiYgV.net
a, b, c > 0
1 + 3/(ab+bc+ca) ≧ 6/(a+b+c)

( ゚∀゚) ハァハァ…

594 :132人目の素数さん:2021/02/14(日) 19:07:32.95 ID:EPktPcJq.net
>>593
a, b, c > 0
1 + 3/(ab+bc+ca) ≧ 1 + 9/{(a+b+c)^2} ≧ 6/(a+b+c)

('A`) ウーム…

595 :132人目の素数さん:2021/02/16(火) 19:11:23.08 ID:+p1DAQAM.net
底面を凸四角形ABCDとする四角錐V-ABCDについて、
すべての辺に接し、かつ中心が底面上にあるような球が存在するとする。
このとき、
 VA+VB+VC+VD ≦ AB+BC+CD+DA

[大数2021−2月宿題]

596 :132人目の素数さん:2021/02/16(火) 20:08:40.42 ID:ItHwm5kd.net
球の半径を1とし、球の中心をO、∠OAB、∠OBC、∠OCD、∠ODAをa,b,c,dとすれば
AB+BC+CD+DA=2(cot a+cot b +cot c + cot d)、
VA+VB+VC+VD=cot a + tan a +cot b + tan b +cot c + tan c +cot d + tan d
∴ AB+BC+CD+DA-(VA+VB+VC+VD)
=cot a - tan a +cot b - tan b +cot c - tan c +cot d -tan d
以下a+b+c+d=π/2とcot x-tan xの凸性

597 :132人目の素数さん:2021/02/16(火) 21:52:59.29 ID:1xlHv4Kk.net
大数ネタはご遠慮願いたいわ

598 :132人目の素数さん:2021/02/17(水) 00:07:29.10 ID:pOGUunX7.net
>>593-594
 文献[9] 佐藤(訳)、朝倉書店 (2013)
 演習問題1.86 p.40

599 :132人目の素数さん:2021/02/17(水) 00:14:31.35 ID:mKsTbiN1.net
Π[p:素数]cos(2π/p) > 0

600 :132人目の素数さん:2021/02/17(水) 00:22:20.07 ID:FhoAKrRQ.net
>>596
三行目では, VA=cot a + tan a になるってことですか?
tan a の出所がわからないです。

601 :132人目の素数さん:2021/02/17(水) 01:31:32.48 ID:3OYBmM8w.net
あ、しまった>>596は撤回です

602 :132人目の素数さん:2021/02/18(木) 15:24:16.44 ID:gfQsTfgF.net
内接円をもつ四角形ABCDで、内接円の中心を I とするとき

(IA+ID)^2+(IB+IC)^2 ≦ (AB+CD)^2

603 :132人目の素数さん:2021/02/23(火) 15:53:16.27 ID:mfVhACbJ.net
>>599
cos(π) = - 1,
cos(2π/3) = - 1/2,
cos(2π/5) = (√5 -1)/4 = 1/(2φ) = 0.309017
cos(2π/7) = 0.6234898
Π[p≧11] cos(2π/p) > Σ[p≧11] (1 - 2ππ/p^2)
  > 1 - Σ[p≧11] 2ππ/p^2
  > 1 - (2ππ)Σ[k≧5] 1/(2k+1)^2
  = 1 - (2ππ)(ππ/8 - 1 - 1/9 - 1/25 - 1/49 - 1/81)
  = 1 - (2ππ)・0.0498356
  = 0.01628475
∴ (与式) > 0

なお
 Π[p≧7] cos(2π/p) = 0.3338
 Π[p≧11] cos(2π/p) = 0.5354
(与式) ≒ 0.05158

604 :132人目の素数さん:2021/02/23(火) 16:29:58.81 ID:mfVhACbJ.net
Π[p≧11] cos(2π/p) > Π[p≧11] cos(2π/(p-1))
 > Π[n=5,∞] cos(π/n)
 = (2√2)Π[n=3,∞] cos(π/n)
 = (2√2)・0.114942        (*)
 = 0.3251052

*) 数セミ増刊「数学の問題」第2集, 日本評論社 (1978)
 ●117 によれば
 Π[n=3,∞] cos(π/n) = 0.114942044853296…

605 :132人目の素数さん:2021/03/03(水) 14:15:42.03 ID:SY070HAY.net
(2)
 |cosθ| ≦ 1/2  ⇔  cos(2θ) ≦ -1/2,
 |cosθ| ≧ 1/2  ⇔  cos(2θ) ≧ -1/2,

(4)
 |cosθ| ≦ cos(72)  ⇔  cos(4θ) ≧ cos(72),
 |cosθ| ≧ cos(72)  ⇔  cos(4θ) ≦ cos(72),

[分かスレ466-119]

606 :132人目の素数さん:2021/03/03(水) 14:37:41.92 ID:SY070HAY.net
(2)
 cos(2θ) + 1/2 = 2 [(cosθ)^2 - 1/4],

(4)
 cos(4θ) - cos(72) = 8 [cos(18)^2 - (cosθ)^2] [cos(72)^2 - (cosθ)^2]

∴ |cosθ| ≦ cos(72)  ⇒  cos(4θ) ≧ cos(72),

[分かスレ466-129]

607 :132人目の素数さん:2021/03/06(土) 19:39:49.08 ID:dHW5XVEt.net
(4)
 |cosθ| ≦ cos(72)  ⇒  cos(4θ) ≧ cos(72),
 |cosθ| ≧ cos(72)  ←  cos(4θ) ≦ cos(72),

608 :132人目の素数さん:2021/03/06(土) 19:43:07.41 ID:dHW5XVEt.net
〔問題157〕
x>0, y>0, z>0 ならば
 (x+y)^z + (y+z)^x + (z+x)^y >2,

[分かスレ466-157, 178]

609 :132人目の素数さん:2021/03/06(土) 19:53:41.41 ID:VMjWPceO.net
>>180
そのわかすれの178のレスでx+y,y+z,z+xのうち1以上のものが少なくとも一つあるとしてるけど、それらのケースに帰着できるわけじゃないよね?
最小値はx=y=z=0.184付近だし
単にすぐに除外していいケース述べてるだけだよね?

610 :132人目の素数さん:2021/03/07(日) 06:25:23.53 ID:gfZuqlK8.net
全部1未満のとき
 0 < x, y, z < 1.
 f(z) = a^(1-z) (a>0) は下に凸だから
 f(z) < f(0)(1-z) + f(1)z,  (0<z<1)
 a^(1-z) < a(1-z) + z < a+z,
 a^z > a/(a+z),   … ベルヌーイの不等式
∴ (x+y)^z > (x+y)/(x+y+z),
 巡回的にたす。

611 :132人目の素数さん:2021/03/07(日) 07:55:47.98 ID:hMmQPJhz.net
>>609
なんで上から抑えるん?
最小値>2を示せでしょ?

612 :132人目の素数さん:2021/03/10(水) 06:18:44.76 ID:dMP4wwTf.net
〔問題596〕
正の実数 a,b,c が a+b+c = 1 を満たすとき
 (1/a - a)(1/b - b)(1/c - c) ≧ (3 - 1/3)^3,
 等号成立は a=b=c = 1/3.
を示せ。

[高校数学の質問スレ410-596,599,610]

613 :132人目の素数さん:2021/03/10(水) 22:08:55.30 ID:NWVdDcAf.net
「任意の自然数nに対して、n<2^nが成立」
これを色々な方法で証明せよ

よくある証明方法は帰納法、二項定理の利用、微分によるなどあるが、それ以外もあるんかな

614 :132人目の素数さん:2021/03/11(木) 06:14:13.22 ID:JY2ui+vd.net
帰納法
 2^n = 2^{n-1} + 2^{n-1} > (n-1) + 1 = n,
あるいは
 2^n = 2^{n-1} + 2^{n-2} + ・・・・ + 2 + 1 + 1 ≧ n + 1,
           (n+1)項

a_1, a_2, ・・・・, a_n ≧ 0 のとき
(1+a_1)(1+a_2)・・・・(1+a_n) = 1 + s_1 + ・・・・ + s_n ≧ 1 + s_1,
  s_k は k次の基本対称式
  s_1 = a_1 + a_2 + ・・・・ + a_n,
より
 2^n ≧ 1 + n

615 :132人目の素数さん:2021/03/11(木) 12:04:01.19 ID:qBeEcW7U.net
俺が考えていた証明

n(1/n-1/2^n)=1-n/2^n
=(Σ1/2^i)-n/2^n
>(Σ1/2^i)-(1/2+…+1/2^n)>0
よりn>0だから
1/n-1/2^n>0⇔n<2^n

616 :132人目の素数さん:2021/03/15(月) 02:13:29.47 ID:M36DVxm1.net
不等式ってこういう気まぐれ?なときもあるんだね。。
https://ja.wikipedia.org/wiki/%E3%82%B7%E3%83%A3%E3%83%94%E3%83%AD%E3%81%AE%E4%B8%8D%E7%AD%89%E5%BC%8F

617 :132人目の素数さん:2021/03/15(月) 22:30:10.49 ID:on5dd3Wr.net
>>616
n=3のときの、ネビットの不等式に (;゚∀゚)=3ハァハァ した若い頃が懐かしい…

618 :132人目の素数さん:2021/03/17(水) 08:27:58.15 ID:Rkkg81B/.net
>>612
a+b+c = 1 より
 G = (abc)^{1/3} ≦ 1/3,     (AM-GM)

 1/y - y = (1+y)・(1/y - 1),
より
(1/a - 1)(1/b - 1)(1/c - 1) = (1-a -b -c)/(abc) + (1/a + 1/b + 1/c) - 1
 = 1/a + 1/b + 1/c - 1
 ≧ 3/G - 1
 ≧ 2(1/G + 1)   (G≦1/3)
 = 2(1/3G + 1/3G + 1/3G + 1)
 ≧ (2/(3G)^{1/4})^3,

(1+a)(1+b)(1+c) ≧ (1+G)^3  (コーシー)
 = (1/27)(3G+1+1+1)(1+3G+1+1)(1+1+3G+1)
 ≧ ((4/3)(3G)^{1/4})^3,

辺々掛けて (左辺) ≧ (8/3)^3.

619 :132人目の素数さん:2021/03/17(水) 08:34:06.62 ID:Rkkg81B/.net
〔問題3204〕
a≧b≧c≧d≧0 のとき
 (a+2b) (aa+bb) ≦ (a+b)^3
 (a+2b+3c) (aa+bb+cc) ≦ (a+b+c)^3,
 (a+2b+3c+4d) (aa+bb+cc+dd) ≦ (a+b+c+d)^3,

注) 5文字の場合は aa(b-d-2e) が出て来ます…orz

すうじあむ
 http://suseum.jp/gq/question/3204

620 :132人目の素数さん:2021/03/18(木) 01:32:04.79 ID:iElyCuOB.net
>>616
こういうのもなんか理由があるっぽい

621 :132人目の素数さん:2021/04/08(木) 19:31:57.99 ID:jAHOCp/v.net
〔例2.4.6〕
三角形の辺の長さを a,b,c, 面積を凾ニすると
  ≦ (3/4)abc/√(aa+bb+cc),

佐藤(訳), 文献9, 朝倉書店 (2013)  p.89

622 :132人目の素数さん:2021/04/08(木) 20:11:29.99 ID:jAHOCp/v.net
(略証)
  = (1/4)√{4(aabb+bbcc+ccaa) - (aa+bb+cc)^2}  (Heron)
  = (1/4)√{4(xy+yz+zx) - (x+y+z)^2}
  ≦ (1/4)√{9xyz/(x+y+z)}      (Schur-1)
  = (3/4)abc/√(aa+bb+cc),

* Schur-1
 F_1(x,y,z) = (x+y+z)^3 - 4(x+y+z)(xy+yz+zx) + 9xyz
  = x(x-y)(x-z) + y(y-z)(y-x) + z(z-x)(z-y) ≧ 0,

623 :132人目の素数さん:2021/04/24(土) 23:51:27.16 ID:nN6RssoR.net
Twitterで拾った問題

https://i.imgur.com/UT7t9WF.jpg

624 :132人目の素数さん:2021/04/25(日) 04:42:31.91 ID:We8cr6tt.net
〔問題〕
正の実数 a,b,c について、次が成り立つことを示せ。
 {aa(b+c)+4}/(a+2)^3 + {bb(c+a)+4}/(b+2)^3 + {cc(a+b)+4}/(c+2)^3 ≧ 2/3.
等号成立は (a,b,c) = (1,1,1) のとき

625 :132人目の素数さん:2021/04/25(日) 05:46:51.43 ID:We8cr6tt.net
(略証)
{aa(b+c) + 2 + 2} / (a+1+1)^3
 ≧ 1/ {(1+1+1)[a/(b+c) + 1/2 + 1/2)]}  (← コーシー)
 = (b+c) / {3(a+b+c)},
巡回的にたす。

626 :132人目の素数さん:2021/04/25(日) 10:47:03.87 ID:2luW0iZY.net
Twitterから拾った問題

https://i.imgur.com/MTQJIvZ.jpg

627 :132人目の素数さん:2021/04/25(日) 11:03:19.13 ID:YeL676w3.net
>>625
作問者の天真(Twitter:@bon_miss_tenma)です
こんなにあっさり解かれるとは思ってませんでしたw
ついでに620さんの問題も僕のだったりします、是非挑戦してください!

628 :132人目の素数さん:2021/04/25(日) 19:29:46.14 ID:We8cr6tt.net
〔問題620〕
正の実数 a,b,c が aa+bb+cc=3 を満たすとき、次を示せ。
 (2a+1)/(b+c+1)^3 + (2b+1)/(c+a+1)^3 + (2c+1)/(a+b+1)^3 ≧ 1/3,
等号成立は (a,b,c)=(1,1,1) のとき。

(略解)
 (左辺) ≧ (b+c+1)/(b+c+1)^3 + (c+a+1)/(c+a+1)^3 + (a+b+1)/(a+b+1)^3
  = 1/(b+c+1)^2 + 1/(c+a+1)^2 + 1/(a+b+1)^2  (← チェビシェフ)
  ≧ 9/{(b+c+1)^2 + (c+a+1)^2 + (a+b+1)^2}  (← AM-HM / コーシー)
  ≧ 3/{(bb+cc+1) + (cc+aa+1) + (aa+b+1)}
  = 3/{2(aa+bb+cc)+3}
  = 1/3,      (← 題意)

629 :132人目の素数さん:2021/04/26(月) 02:25:57.94 ID:y9M7sTQu.net
(補足)
チェビシェフで
(a+1/2)/(b+c+1)^3 + (b+1/2)/(c+a+1)^3 - (a+1/2)/(c+a+1)^3 - (b+1/2)/(b+c+1)^3
 = (a-b) {1/(b+c+1)^3 - 1/(c+a+1)^3}
 ≧ 0,
循環的にたすと
 (左辺) - 1/(b+c+1)^2 - 1/(c+a+1)^2 - 1/(a+b+1)^2 ≧ 0,

630 :132人目の素数さん:2021/04/28(水) 04:23:14.87 ID:B9p/ERZg.net
>>624
>>626

〔問題34〕
a,b,c > 0 のとき
 (a(b+c)+1)/(b+c+1)^2 + (b(c+a)+1)/(c+a+1)^2 + (c(a+b)+1)/(a+b+1)^2 ≧ 1,
 Inequalitybot [34] ☆5
 JMO-2010 問4

Inequalitybot も問題番号で検索できるようになってます。

631 :132人目の素数さん:2021/04/28(水) 07:27:45.63 ID:B9p/ERZg.net
>>589
〔問題48〕
a,b,c >0 のとき
 (a^5-a^2+3)(b^5-b^2+3)(c^5-c^2+3) ≧ (a+b+c)^3

 USAMO-2004, Q5
 Inequalitybot [48] ☆6

632 :132人目の素数さん:2021/04/28(水) 16:58:32.60 ID:B9p/ERZg.net
>>619
 (a+b)^3 - (a+2b)(aa+bb) = aab + (2a-b)bb ≧ 0,
 (a+b+c)^3 - (a+2b+3c)(aa+bb+cc) = aab + (2a-b)bb + (2a+b-2c)cc + 6abc ≧ 0,
 (a+b+c+d)^3 - (a+2b+3c+4d)(aa+bb+cc+dd)
= aa(b-d) + (2a-b-d)bb + (2a+b-2c-d)cc + (2a+b-3d)dd + 6(abc+abd+acd+bcd) ≧ 0,

633 :132人目の素数さん:2021/04/28(水) 17:05:44.79 ID:B9p/ERZg.net
これと以下を組み合わせた問題があった。
〔補題〕
a+b+c+… = 1 のとき
  (a^a)(b^b)… ≦ (aa+bb+…),
(略証)
a+b+c+… = s とおく。
y=log(x) は上に凸だから Jensen で
 a・log(a) + b・log(b) + ・・・・ ≦ s・log((aa+bb+・・・・)/s)
 (a^a)(b^b)… ≦ {(aa+bb+…)/s}^s,
s=1 とおく。

634 :132人目の素数さん:2021/04/28(水) 19:31:51.11 ID:B9p/ERZg.net
〔問題〕
 tan(1/2) > cos(1).
これの証明はどうすれば出来ますか?

 高校数学の質問スレ411- 028, 936

635 :132人目の素数さん:2021/04/28(水) 23:28:33.43 ID:Tu1Xrn91.net
t = tan(1/2)とおいて
tan(1/2)-cos(t)=(t^3+t^2+t-1)/(t^2+1)
なのでコレが+を言えば良い
tan(1/2)=0.546302.....
t^3+t^2+t-1は単調増大で0になるのはt=0.543689....
とりあえず5次までマクローリン展開して
tan(1/2)
>1/2+(1/3)(1/2)^3+(2/15)/(1/2)^5=131/240=0.54583333......

636 :132人目の素数さん:2021/04/29(木) 02:03:41.02 ID:mxa1BnUU.net
>>634
θ = 1/2 とおいて
tanθ - cos(2θ) = tanθ - 1 + 2(sinθ)^2
 = tanθ - 3/2 + {1/2 + 2(sinθ)^2}
 ≧ tanθ - 3/2 + 2sinθ  (AM-GM)
 = tanθ + 2sinθ - 3θ
 ≧ 0,   (Snellius-Huygensの式)

637 :132人目の素数さん:2021/04/29(木) 02:45:36.56 ID:NbeeKPJA.net
このスネル・ホイヘンスの不等式、以前からどうやって見つけたのか気になってるヤツだ

638 :132人目の素数さん:2021/04/30(金) 23:51:02.05 ID:nccWEVYf.net
>>636
上手だなあ

639 :132人目の素数さん:2021/05/01(土) 14:45:07.43 ID:kM6Q2nUa.net
数学コンテストの問題から二つの不等式問題を拾ってみた

数オリ中国代表選抜の問題
https://i.imgur.com/fDeESFB.jpg

数オリ米国代表選抜の問題
https://i.imgur.com/t1tus8h.jpg

640 :132人目の素数さん:2021/05/01(土) 15:17:41.91 ID:34KV0J3x.net
>>639
さいきん、関数不等式に(;゚∀゚)=3ハァハァでござる

641 :132人目の素数さん:2021/05/04(火) 23:49:09.08 ID:+H9X9UVo.net
a,b,c > 0 に対して、
(a+b+c)^3 ≧ 27abc{(aa+bb+cc)/(ab+bc+ca)}^(25/27)

642 :132人目の素数さん:2021/05/05(水) 05:39:50.22 ID:16g2LNeV.net
〔簡易版〕
a,b,c>0 に対して
 (a+b+c)^3 ≧ 27abc{(aa+bb+cc)/(ab+bc+ca)}^(2/3).

(略証)
(a+b+c)^6 = {(aa+bb+cc) + (ab+bc+ca) + (ab+bc+ca)}^3
  ≧ 27(aa+bb+cc)(ab+bc+ca)^2,    (AM-GM)
2/3 乗して
(a+b+c)^4 ≧ 9(ab+bc+ca)^2 {(aa+bb+cc)/(ab+bc+ca)}^(2/3)
  ≧ 27(a+b+c)abc {(aa+bb+cc)/(ab+bc+ca)}^(2/3),

元の問題は解けぬwww

643 :132人目の素数さん:2021/05/05(水) 05:41:42.08 ID:16g2LNeV.net
〔問題3.85〕
実数a,b,cに対して
 (aa+2)(bb+2)(cc+2) ≧ 3(a+b+c)^2,

 APMO-2004 A5.改
 文献[9] 佐藤(訳)、朝倉書店 (2013) 問題3.85 p.140
 Inequalitybot [20] ☆8
 [高校数学の質問スレ412−029,036,040]

644 :132人目の素数さん:2021/05/05(水) 05:49:27.56 ID:16g2LNeV.net
(解1)
(aa+2)(bb+2)(cc+2) - 3(a+b+c)^2
 = (1/3){(aa+5)(bc-1)^2 + (bb+5)(ca-1)^2 + (cc+5)(ab-1)^2
  + (ab+bc+ca-3)^2 + (a-b)^2 + (b-c)^2 + (c-a)^2}
 ≧0

(解2)
 (aa+2)(bb+2)(cc+2) - 3(a+b+c)^2
 = aa + bb + cc + 2abc + 1 - 2(ab+bc+ca)
 + (abc-1)^2
 + 2(ab-1)^2 + 2(bc-1)^2 + 2(ca-1)^2,

 文献[9] の演習問題1.90 (ii) p.41-42 に帰着する。
〔問題1.90〕(ii)
a,b,c を非負実数とする。このとき
 aa + bb + cc + 2abc + 1 ≧ 2(ab+bc+ca),

645 :132人目の素数さん:2021/05/05(水) 13:18:52.39 ID:+30DA6pc.net
>>641-642
ネットで拾ったんだけど、答えがないな
https://mathifc.wordpress.com/2012/09/20/inequality-106/

646 :132人目の素数さん:2021/05/06(木) 05:59:11.34 ID:Vi6k/Ft1.net
>>644
〔例題2.1.11〕
(7) a,b,c が非負実数のとき
 aa + bb + cc + 2abc + 1 ≧ 2(ab+bc+ca),


文献[8] 安藤, 数学書房 (2012) p.36

647 :132人目の素数さん:2021/05/30(日) 05:50:26.45 ID:EIfW8DuI.net
>608
{x+y, y+z, z+x} のうち1以上のものが

・2個以上のときは 明らか。
・1個以下のときは 1 > y+z, z+x より 0 < x, y, z < 1  >>610

648 :132人目の素数さん:2021/05/30(日) 06:56:34.75 ID:EIfW8DuI.net
>>637
マクローリン展開
 sinθ = θ - (1/3!)θ^3 + (1/5!)θ^5 - (1/7!)θ^7 + (1/9!)θ^9 - …
 tanθ = θ + (1/3)θ^3 + (2/15)θ^5 + (17/315)θ^7 + (62/2835)θ^9 + …
から思い付いたのかも。

>>102 にもあるよ。
 H = θ - (1/180)θ^5 - (1/1512)θ^7 - (1/25920)θ^9 - …
 G = θ + (1/45)θ^5 + (4/567)θ^7 + (1/405)θ^9 + …
 A = θ + (1/20)θ^5 + (1/56)θ^7 + (7/960)θ^9 + …
 A + H - 2G = (1/324)θ^7 + (1/432)θ^9 - …
 AH/GG = (2cosθ+1)/{(2+cosθ)(cosθ)^(1/3)}
    = 1 + (1/324)θ^6 + (1/648)θ^8 + …

649 :132人目の素数さん:2021/05/30(日) 08:38:14.15 ID:EIfW8DuI.net
ついでに…
s>0, t>0 とし
 A = (s+s+t)/3,
 G = (sst)^(1/3)
 H = 3st/(s+t+t),
とおくと
 H < G < A,
 AH > GG,   (0<s<t)
 A+H > 2G,   (0<s<t)
(略証)
 AH = (s+s+t)st/(s+t+t),
 G^3 = sst,
より
 (AH)^3 - G^6 = tt {t(s+s+t)^3 - s(s+t+t)^3}{s/(s+t+t)}^3
  = tt(s+t){(t-s)s/(s+t+t)}^3 > 0,
∴ AH > GG,

 (A+H)/2 = (ss+7st+tt)/[3(s+t+t)],
 G^3 = sst,
より
 {(A+H)/2}^3 - G^3 = {(t-s)^3 + 27stt}{(t-s)/[3(s+t+t)]}^3 > 0,
∴ A+H > 2G,

650 :132人目の素数さん:2021/05/30(日) 18:54:47.33 ID:SMLQU2Ye.net
>>648
テイラー展開は、あまり時代に合わんような気もする。まあ古くから、特殊な場合だけや結果だけ知られているということがよくあるのと、詳しくないので結論付けられない。

ホイヘンスによる証明があったわ。
円の大きさの発見 : 1654年ホイヘンスによる円周率の計算
https://www2.tsuda.ac.jp/suukeiken/math/suugakushi/sympo27/27_tanuma.pdf
(近似)式自体は、15世紀のニコラウス・クザーヌスまで遡れるらしい。

グレゴリーやニュートンが17世紀後半にべき級数展開したらしいから、ホイヘンスは知らないような気もする。代数計算得意じゃないとキツイし。

651 :132人目の素数さん:2021/06/06(日) 02:15:52.79 ID:Q09kUO/y.net
1992年IMO5番
https://i.imgur.com/NvlwAw4.png

652 :132人目の素数さん:2021/06/08(火) 05:51:17.44 ID:Hilnv+E/.net
5.Sは3次元座標空間の有限個の点の集合である。
S_x, S_y, S_z はそれぞれ、Sの点の yz-平面, zx-平面, xy-平面への正射影からなる点の集合である。
次を証明せよ。
  | S |^2 ≦ |S_x|・|S_y|・|S_z|
ここに | A | は有限集合Aの要素の個数である。

653 :132人目の素数さん:2021/06/08(火) 05:57:08.83 ID:Hilnv+E/.net
>>284

f(x)は下に凸な関数とする。自然数nに対して不等式
 nΣ[k=0,n] f(2k) > (n+1)Σ[k=1,n] f(2k-1)
を示せ。

[面白スレ36.256-260]

654 :132人目の素数さん:2021/06/08(火) 20:03:58.37 ID:Hilnv+E/.net
>>651 >>652
z値の集合を {z1, …, zi, …, zn} とする。
S, Sy, Sx の点を z値で分類する。
S, Sy, Sx の点のうち z=zi をみたすものの個数を |Li|, ai, bi とする。

(1) |Li| ≦ ai・bi,

(2) |S| = |L1| + … + |Li| + … + |Ln|,

(3) |Sy| = a1 + … + ai + … + an,
  |Sx| = b1 + … + bi + … + bn,

(4) |Li| ≦ |Sz|,

(1) と (4) を掛けて
 |Li|^2 ≦ (ai・bi) |Sz|,
 |Li| ≦ √(ai・bi) √|Sz|,    ・・・・ (5)

(2), (5) より
|S|^2 ≦ {√(a1・b1) + … + √(ai・bi) + … + √(an・bn)}^2・|Sz|
  ≦ (a1 + … + ai + … + an)(b1 + … + bi + … + bn)|Sz|   コーシー
  = |Sy| |Sx| |Sz|,

http://www.youtube.com/watch?v=IzitrvYnNkc 11:08,

655 :132人目の素数さん:2021/06/12(土) 15:46:26.57 ID:rM3zqrZ1.net
1993年日本数学オリンピック本選5番

https://i.imgur.com/kwA3sGg.png

656 :132人目の素数さん:2021/06/14(月) 16:23:08.26 ID:lFKMSNRU.net
0<k≦3, a,b,c>0のとき
3-k+k(abc)^(2/k)+a^2+b^2+c^2≧2(ab+bc+ca)
を示せ

657 :132人目の素数さん:2021/06/15(火) 01:00:18.66 ID:78D2HNY3.net
>>646
>>656
似た問題くれ

658 :132人目の素数さん:2021/06/15(火) 20:42:53.54 ID:iDQ7MEu/.net
>>656
0<k≦3 ゆえ x^(3/k) は下に凸。 x=1 で接線を曳いて、
 (3-k) + k・x^(3/k) ≧ 3x,
(左辺) - (右辺) ≧ aa+bb+cc - 2(ab+bc+ca) + 3(abc)^(2/3)
 ≧ aa+bb+cc - 2(ab+bc+ca) + 9abc/(a+b+c)    (AM-GM)
 = F1(a,b,c)/(a+b+c)
 ≧ 0,

*) Schurの不等式
F1(a,b,c) = a(a-b)(a-c) + b(b-c)(b-a) + c(c-a)(c-b)
 = (a+b+c)^3 - 4(a+b+c)(ab+bc+ca) + 9abc ≧ 0.

659 :132人目の素数さん:2021/06/17(木) 14:58:41.31 ID:NM0vuEzL.net
>>658
3/kではなくて2/kだぞ

660 :132人目の素数さん:2021/06/17(木) 20:08:44.51 ID:lnjH0V31.net
x = (abc)^(2/3)

661 :132人目の素数さん:2021/07/24(土) 10:02:29.51 ID:olJMYtps.net
IMO2021の2番に不等式問題出たな

https://i.imgur.com/3822PyY.png

662 :132人目の素数さん:2021/07/24(土) 12:58:46.19 ID:iF34JJ+s.net
>>661
ウホッ、いい不等式!

663 :132人目の素数さん:2021/07/24(土) 13:52:58.43 ID:iF34JJ+s.net
a,b,c > 0、ab+bc+ca+abc=4 のとき、a+b+c ≧ ab+bc+ca.
ベトナム1996らしい

664 :132人目の素数さん:2021/07/25(日) 04:46:50.40 ID:0rv1EuHc.net
(略解)
 t = ab+bc+ca < 3,
と仮定すると
 u = abc < 1,   (AM-GM)
となり題意に反する。
∴ 3 ≦ t < 4,
∴ s = a+b+c ≧ tt/3 ≧ 3,

(s-t)(ss+st+tt - 4t)
 = (4-t)(t-3)(t+3) + (s^3-4st+9u)
 = (4-t)(t-3)(t+3) + F1(a,b,c)
 ≧ 0,   (← Schur-1)
∴ s-t ≧ 0,
 [面白スレ37.704] にもあった。

665 :132人目の素数さん:2021/07/25(日) 04:52:39.17 ID:0rv1EuHc.net
訂正スマソ
 s = a+b+c ≧ √(3t) ≧ 3,   (AM-GM)

666 :132人目の素数さん:2021/07/25(日) 05:08:13.86 ID:0rv1EuHc.net
〔類題184〕
a,b,c>0, a+b+c+abc=4 のとき a+b+c≧ab+bc+ca,

大数宿題 2010-Q7
[不等式スレ7.114-115,160]
Inequalitybot [184] ☆7

667 :132人目の素数さん:2021/07/25(日) 05:12:18.46 ID:0rv1EuHc.net
(略解)
s = a+b+c < 3 と仮定すると
 u = abc < 1   (AM-GM)
となり題意に反する。
∴ 3 ≦ s < 4.

4s(s-t) = (4-s)(s-3)(s+3) + 9(4-s-u) + (s^3 -4st +9u)
  = (4-s)(s-3)(s+3) + 9(4-s-u) + F1(a,b,c)
  ≧0,     (← Schur-1)
∴ s-t ≧ 0.

668 :132人目の素数さん:2021/07/25(日) 06:48:26.36 ID:0rv1EuHc.net
>>661
〔問題2.〕
 任意の実数 x1, x2, ・・・・, xn に対して、不等式
     Σ[i=1,n] Σ[j=1,n] √|xi-xj| ≦ Σ[i=1,n] Σ[j=1,n] √|xi+xj|,
が成り立つことを示せ。

669 :132人目の素数さん:2021/07/25(日) 11:16:51.60 ID:236oCq6r.net
実質極値がa=b=cの時でしかもそれが未定定数法で簡単に求まるやつはなんかもひとつやな

670 :132人目の素数さん:2021/07/25(日) 18:35:50.30 ID:0rv1EuHc.net
>>664
 s≧3 どこにも使ってないの、なんかもひとつやな

671 :132人目の素数さん:2021/07/27(火) 19:25:05 ID:gRrHTwn5.net
>>661
こんな良い不等式がまだ残ってるとは

これルートなくても成り立ちそうだけど、その場合は簡単に示せたりする?

672 :132人目の素数さん:2021/07/29(木) 01:08:01 ID:gaBM8HMZ.net
複素数 z (0≦arg(z) < 2π) に対して、
   |z-1| < |z| - 1 + |z|*arg(z).
( ゚∀゚) ウヒョッ!

673 :132人目の素数さん:2021/07/29(木) 01:09:26 ID:gaBM8HMZ.net
>>672
条件に |z|>1 を追加。

674 :132人目の素数さん:2021/07/29(木) 21:13:16 ID:a7gJLkin.net
a_1≧a_2≧…≧a_n>0かつa_1+a_2+…+a_n=1のとき
a_1+2a_2+…na_nのとりうる値の範囲を求めよ.

675 :132人目の素数さん:2021/07/30(金) 06:22:01 ID:oWjQc2j0.net
f(a) = Σ[k=1,n] k・a_k とおく。
f(1, 0, …, 0) = 1 (最小)
f(1/n, 1/n, …, 1/n) = (n+1)/2 (最大)

(略証)
f(a) - 1 = (a_1+a_2+…+a_n - 1) + Σ[k=2,n] (k-1) a_k ≧ 0,
(n+1)/2 - f(a) = Σ[k=1,n] ((n+1)/2 - k) a_k
   = Σ[k'=1,n] (k' - (n+1)/2) a_{n+1-k'}
   = (1/2)Σ[k=1,n] ((n+1)/2-k) (a_k - a_{n+1-k})  (←同符号)
   ≧ 0,

676 :132人目の素数さん:2021/07/30(金) 06:46:40 ID:oWjQc2j0.net
>>672
?不等式より
 |z - 1| ≦ ||z| - 1| + |z - |z|| < ||z| - 1| + |z|・arg(z),

677 :132人目の素数さん:2021/07/30(金) 14:02:27 ID:oWjQc2j0.net
>>675
(n+1)/2 - f(a) = ((n+1)/2) (1 - a_1 - a_2 - … - a_n)
      + (1/2) Σ[k=1,n-1] k(n-k) (a_k - a_{k+1})
     ≧ 0,
の方がいいか…

678 :132人目の素数さん:2021/07/31(土) 10:02:06 ID:aE5MLnpD.net
有名不等式が大量に載ってるpdfを発見したので載せてみる
http://www.lkozma.net/inequalities_cheat_sheet/ineq.pdf

あと2020年度imoショートリストから

https://i.imgur.com/ju1Dyee.png

679 :132人目の素数さん:2021/08/17(火) 13:07:12 ID:3+UNf6gr.net
>>678
グッジョブ
分割して日替わり壁紙にしよう

680 :132人目の素数さん:2021/08/26(木) 20:07:00 ID:hGc0TLQT.net
eと(1+1/n)^nが登場する不等式をたくさんください

681 :132人目の素数さん:2021/09/04(土) 18:23:48 ID:KMsJe/e+.net
a, b, c が0以上かつ a^2 + b^2 + c^2 = 1 を満たすとき,
(a+bーc)^n + (b+c-a)^n + (c+a-b)^n (n は3以上の整数) 
の最大値と最小値を求めよ.

682 :132人目の素数さん:2021/09/04(土) 21:02:15 ID:HGuBdRDo.net
最大値 2^{n/2}
  a = 0, b = c = 1/√2 など。 (x=√2, y=z=0, etc.)
最小値 (1/3)^{n/2 - 1}
  a = b = c = 1/√3,  (x=y=z=1/√3)

x = b+c-a, y = c+a-b, z = a+b-c とおくと
1 = aa + bb + cc
 = {(x+y)^2 + (y+z)^2 + (z+x)^2}/4
 = {(x+y+z)/√3}^2 + (1/4){(x-y)/√2}^2 + (1/4){(x+y-2z)/√6}^2,
回転楕円体 (どら焼き形)
 短軸:1  (1,1,1)方向  
 長軸:2  それと垂直方向

683 :132人目の素数さん:2021/09/04(土) 23:49:24 ID:KMsJe/e+.net
>> 676

もう少し具体的に

684 :132人目の素数さん:2021/09/05(日) 02:00:08 ID:HFxHmzMl.net
a=u^2,b=v^2,c=w^2
束縛
C = u^4+v^4+w^4-1
評価関数
S = (v^2+w^2-u^2)^n+(w^2+u^2-v^2)^n+(u^2+v^2-w^2)^n

s = 2n((v^2+w^2-u^2)^(n-1)+(w^2+u^2-v^2)^(n-1)+(u^2+v^2-w^2)^(n-1))とおいて
dC=4(u^3,v^3,w^3)
dS=s(u,v,w)
s≠0により
dSがdCで張られる
⇔vw(v^2-w^2)=wu(w^2u^2)=uv(u^2-v^2)=0
⇔u^2=v^2=w^2 or u^2=v^2 & w=0 or u=v=0 or...

685 :132人目の素数さん:2021/09/05(日) 06:18:37 ID:Zhg5gGCb.net
専門的過ぎてついていけない
数オリの高校生の理解できる解法でお願いします

686 :132人目の素数さん:2021/09/05(日) 10:01:38 ID:HFxHmzMl.net
数学の問題は進んだテクニック使っても全然簡単にならず、実は中学生でも理解できるような話の方が楽に解ける時がある
数オリとかの問題とかそういう問題のオンパレードだし、ピーターフランクルとかそんな問題大好きの人もいっぱいいる
しかしそれは進んだ数学を勉強しないでいい理由になどにはならないし、ましてや逆に言えば、進んだテクニック使えば楽に解ける問題をいつまでもいつまでもそういう”初頭数学縛り”をかけて解くのは単なる“自己満”でしか無い
不等式の話を本当に極めるなら未定乗数法は絶対避けては通れない

687 :132人目の素数さん:2021/09/05(日) 12:53:40 ID:LDbpAA38.net
grad(f(u,v,w)) = ∇f = (∂f/∂u, ∂f/∂v, ∂f/∂w)
s1 = 2n{-(v^2+w^2-u^2)^(n-1) + (w^2+u^2-v^2)^(n-1) + (u^2+v^2-w^2)^(n-1)},
s2 = 2n{(v^2+w^2-u^2)^(n-1) - (w^2+u^2-v^2)^(n-1) + (u^2+v^2-w^2)^(n-1)},
s3 = 2n{(v^2+w^2-u^2)^(n-1) + (w^2+u^2-v^2)^(n-1) - (u^2+v^2-w^2)^(n-1)},
とおくと
 grad(C) = ∇C = 4(u^3, v^3, w^3)
 grad(S) = ∇S = (s1・u, s2・v, s3・w)
       = s(u, v, w)
ここから ついていけない…

688 :132人目の素数さん:2021/09/06(月) 18:13:36 ID:eC9BaMcK.net
問題[2]
 a_n = (1 + 1/n)^n, b_n = (1 + 1/n)^(n+1)  (nは正の整数)
とおくとき、nが増加するとa_nは増加し、b_nは減少することを証明せよ。
 (数学検定 2011年秋, 1級 2次 問題[2] の一部)
* 作問者は AM-GM を活用する解答を期待していたが…

〔補題258〕               >>262
 (1) (1 + 1/n)^(n+1/2) は単調減少でeに収束
 (2) n! < n^(n+1/2) / e^(n-1),
 (3) (2n)! / n! < (√2)(4n/e)^n,

>>267
Σ[k=1,n] (1/((k+1)(k!)^2))^(1/k) ≒ 1.99877613 - ee/n + 64.5/nn - …

689 :132人目の素数さん:2021/09/15(水) 06:12:14 ID:UyKWpegQ.net
>>680
〔モローの不等式〕
 {2n/(2n+1)}e < (1+1/n)^n < {(2n+1)/(2n+2)}e,

左側は 補題(1) より
 {2n/(2n+1)}e < 1/√(1+1/n)・e < (1+1/n)^n

http://www.youtube.com/watch?v=FDTaIYjWR2E 20:24,
数セミ増刊「数の世界」日本評論社 (1982) p.82

690 :132人目の素数さん:2021/09/15(水) 15:24:37 ID:+P/8oXVv.net
x_1,x_2,...,x_n>0, Πx_k=1のとき次を示せ
Σ1/(n-1+x_k)≦1

691 :132人目の素数さん:2021/09/15(水) 17:00:11 ID:cOPYG12B.net
f(t)=1/(n+e^t)、F(t1,‥) = Σf(ti)とおく
f(t)はt≧lognで下に凸かつt≦lognで上に凸
全てtiがlognより小さい領域ではti=0のときFは最大値1
そうでない領域でΣti=0かつF(ti)>1が存在すれば
t1 =(n-1)c, ti=-c (i≧2,t1>logn)
であるtiで存在する
e^t=uとおいて
F(ti)-1
= 1/(n-1+u^(n-1) + (n-1)/(n-1+1/u)-1
= 1/(n-1+u^(n-1) - 1/((n-1)u+1)
しかしu≧1において
n-1+u^(n-1)≧(n-1)u+1
であるから矛盾

692 :132人目の素数さん:2021/09/16(木) 05:07:13 ID:Sn49tAbo.net
背理法で…
不等式が成り立たないとする。すなわち、
 Σ[k=1,n] 1/(n-1+x_k) >1,
であると仮定する。このとき
1/(n-1+x_i) > 1 - Σ[k≠i] 1/(n-1+x_k)
 = (1/(n-1))Σ[k≠i] x_k /(n-1+x_k)
 ≧ ( Π[k≠i] x_k /(n-1+x_k) )^{1/(n-1)}, (AM-GM)
となる。i=1,…,n で掛けて
 Π[i=1,n] 1/(n-1+x_i) > Π[k=1,n] x_k /(n-1+x_k),
となるが、これは 1 > Π[k=1,n] x_k を意味するので矛盾である。

ルーマニアMO-1999,
文献[9], 佐藤(訳), 朝倉書店(2013), 問題3.35 p.131
Inequalitybot [109]

693 :132人目の素数さん:2021/09/16(木) 05:13:59 ID:Sn49tAbo.net
〔類題〕
 x_1, x_2, …, x_n >0 が Σ[k=1,n] 1/(n-1+x_k) = 1 を満たすとする。
このとき
   Π[k=1,n] x_k ≧ 1,
を証明せよ。

文献[9], 佐藤(訳), 朝倉書店(2013), 問題1.46改 p.14

694 :132人目の素数さん:2021/09/20(月) 10:34:26 ID:YMP5Sl+4.net
(1)
z,w∈C、|z|=|w|=1 のとき、
|z+1| + |w+1| + |zw+1| ≧ 2

(2)
a,b,c∈C に対して、
|a| + |b| + |c| ≦ |a+b-c| + |b+c-a| + |c+a-b|

( ゚∀゚) ウヒョッ!

695 :132人目の素数さん:2021/09/20(月) 22:52:38 ID:lYkiWXwV.net
(1)は簡単やな
x^をxの複素共役として
|z+1|+|w+1|+|zw+1|
=|z+1|+|w^+1|+|z+w^|
なので|a|+|b|+|c|=1のとき
|b+c|+|c+a|+|a+b|≧2
を示せば良い
b = c exp(2iA), c = exp(2iB), a = exp(2iC), A+B+C=π
となる非負実数A,B,Cがとれるとしてよくこのとき
|b+c|+|c+a|+|a+b|
=2(cosA+cosB+cosC)
であるからcos(x)の凸性により(A,B,C)=(-π,π,π),(π,-π,π),(π,π,-π)のとき最小値2

696 :132人目の素数さん:2021/09/20(月) 23:02:12 ID:lYkiWXwV.net
(2)は力技で
sを複素定数としC^3の領域
R={ .. | a + b + c = 2s }
におけるS=2( |s-a| + |s-b| + |s-c| ) - ( |a| + |b| + | c| )の最小値が0以上であることを示せば良い
それには全微分できない領域で非負、全微分可能な極値で非負を言えば十分
s=0であればS=|a|+|b|+|c|となり自明だからs≠0とする

i) a=0のとき
S=2(|b+c|/2 + |b-c|/2 × 2) - (|b|+|c|)
=|b+c|/2-|b|+|b+c|/2-|c|+|b-c|
≧-|b-c|/2 × 2 + |b-c| = 0

(ii) a=s のとき
このときs=a=b+cより
S=2(|b|+|c|)-(|b+c|+|b|+|c|)
=|b|+|c|-|b+c|≧0

(iii) a=bのとき
このときs=a+c/2より
S=2(|c/2|+|c/2|+|a-c/2|)-(|a|×2+|c/2|)
=|c|+|2a-c|-|2a|≧0

(iv)a,b,cが同一直線上のとき
a,b,cは実数としてよくSをaの関数として見たときlim[a→±∞]S=∞だから極値だけ考えればよく、極値をとるのはa=s,0の場合のみであるから既出の場合に還元される

(v)その他の場合
Sは全微分可能でありz^を複素共役としてe(z)=z/|z|とおけば
dS = -2(e(s-a)^da + e(s-a)da^+ e(s-b)^db +e(s-b)db^+ e(s-c)^dc + e(s-c)dc^)-(e(a)^da+e(a)da^+e(b)^db+e(b)db^+e(c)^dc+e(c)dc^)
でありコレがda+db+dcの複素定数倍であるから
2e(s-a)+e(a)=2e(s-b)+e(b)=2e(s-c)+e(c)=0
である
よってa,b,cが同一直線上となるので既出のケースに還元される

697 :132人目の素数さん:2021/09/21(火) 12:09:47 ID:AENcTZtD.net
>>695
(1)
|a| = |b| = |c| = 1 のとき
 |b+c| + |c+a| + |a+b| ≧ 2,
ですか。

>>696
(2) は簡単やな。Ravi変換で
 b+c-a = p,
 c+a-b = q,
 a+b-c = r,
とおけば
(左辺) = |a| + |b| + |c|
 = |q+r|/2 + |r+p|/2 + |p+q|/2
 ≦ |p| + |q| + |r|.

698 :132人目の素数さん:2021/09/21(火) 12:22:40 ID:IIHpCqtI.net
あれ?
その方法最初に考えてダメと思ったんやけど勘違いしたかな?
まぁ複素係数の微分形式の復習になったからいいけど

699 :132人目の素数さん:2021/09/21(火) 20:58:30 ID:AENcTZtD.net
>>695
 C ≧ π/2 の場合 (鈍角?) は
 |b+c| + |c+a| + |a+b|
 = 2(|cosA| + |cosB| + |cosC|)
 ≧ 2(cosA + cosB)
 ≧ 2(1 + cos(A+B))   (凸性)
 ≧ 2,        (A+B≦π/2)
ですね。あるいは
 cosA + cosB + cosC
 = 1 + 4sin(A/2)sin(B/2)sin(C/2)   (A+B+C=π)
 ≧ 1,

700 :132人目の素数さん:2021/09/22(水) 19:57:40 ID:K2h4cEAP.net
>>694
(1)は簡単やな
 |z+1| + |w^+1| + |z+w^|
 ≧ |(z+1) + (w^+1) - (z+w^)|
 = 2,

 |b+c| + |c+a| + |a+b|
 ≧ |-(b+c) + (c+a) + (a+b)|
 = 2|a|,
同様にして
 |b+c| + |c+a| + |a+b| ≧ 2 Max{|a|,|b|,|c|}

(2)は簡単やな  >>697

701 :132人目の素数さん:2021/09/22(水) 20:21:10 ID:miCnVfcc.net
>>700
なんでそういう書き方するん?
それ読んだ相手がどういう気持ちになるか考えられへんの?

702 :132人目の素数さん:2021/09/24(金) 22:25:33 ID:lJNbXbJw.net
〔問題〕
 Σ[n=2,∞] 1/n^3 < (1+√5)/16 = 0.2022542486

 (阪大-改)

http://www.youtube.com/watch?v=_zGQfWy9j28 22:05
鈴木貫太郎

703 :132人目の素数さん:2021/09/25(土) 11:52:48 ID:S56dxsDJ.net
1/n^3 = n/n^4
 < n/(nn-1/4)^2
 = {(n+1/2)^2 - (n-1/2)^2}/{2(nn-1/4)^2}
 = (1/2){1/(n-1/2)^2 - 1/(n+1/2)^2}
∴ Σ[n=2,∞] 1/n^3 < 2/9 = 0.222222
ぢゃ出ない・・・・orz

704 :132人目の素数さん:2021/09/28(火) 04:04:41 ID:xRhStcay.net
c>0 かつ z,w∈C のとき、
|z+w|^2 ≦ (1+c)|z|^2 + (1 + 1/c)|w|^2.

昔やったかも? ( ゚∀゚)

705 :132人目の素数さん:2021/09/28(火) 12:43:23 ID:ED+tdwHx.net
|z+w|^2 ≦ (|z| + |w|)^2     (三角不等式)
  = |z|^2 + |w|^2 + 2|z||w|
  = |z|^2 + |w|^2 + c|z|^2 + (1/c)|w|^2 - (|z|√c - |w|/√c)^2
  ≦ |z|^2 + |w|^2 + c|z|^2 + (1/c)|w|^2   (GM-AM)
  = (1+c)|z|^2 + (1+1/c)|w|^2.
等号成立は w = cz のとき。

706 :132人目の素数さん:2021/09/28(火) 12:56:38 ID:ED+tdwHx.net
>>702
 φ = (1+√5)/2 = 1.618034… とおく。 (黄金比)
1/n^3 = n/n^4
 ≦ n/{n^4 - (2/φ-1)^2・(nn-4)}    (n≧2)
 = n/{(nn -2 +4/φ)^2 - nn}
 = n/{(nn -n -2 +4/φ)(nn +n -2 +4/φ)}
 = (1/2){1/(nn-n-2 +4/φ) - 1/(nn+n-2 +4/φ)},
∴ Σ[n=2,∞] 1/n^3 < φ/8 = (1+√5)/16,

707 :132人目の素数さん:2021/09/29(水) 05:31:52 ID:lKJ2KBeg.net
>>704
 |z+w|^2 + |z√c - w/√c|^2 = (1+c)|z|^2 + (1+1/c)|w|^2,
だった希ガス。。。

708 :132人目の素数さん:2021/09/29(水) 09:17:12 ID:lKJ2KBeg.net
>>702
1/n^3 = n/n^4
 < n/{n^4 - (√10 -3)^2(nn-9)}    (n≧3)
 = n/{(nn -9 +3√10)^2 - n^2}
 = n/{(nn -n -9 +3√10)(nn +n -9 +3√10)}
 = (1/2){1/(nn -n -9 +3√10) - 1/(nn +n -9 +3√10)},
∴ Σ[n=2,∞] 1/n^3 < 1/8 + (1+√10)/54 = 0.2020792

709 :132人目の素数さん:2021/09/29(水) 10:19:16 ID:mxWzl1I/.net
>>707
( ゚∀゚) キタコレ!

710 :132人目の素数さん:2021/09/29(水) 11:56:39 ID:mxWzl1I/.net
>>694
(2)の等号成立条件はどうなるんでせうか?

711 :132人目の素数さん:2021/09/30(木) 06:51:58 ID:hn+yThHP.net
arg(a) = arg(b) = arg(c) かつ |a|, |b|,| c| が三角不等式を満たす。
ただし arg(0) は任意の値に等しいとする。

∵ arg(p) = arg(q) = arg(r).

712 :132人目の素数さん:2021/09/30(木) 22:46:47 ID:jz/TtT2s.net
Σ(k=1~2n)2nCk×(1/(n-1)^2)^k>2/(n-1)

↑これ成り立ちそうなんだけど証明浮かばん

713 :132人目の素数さん:2021/09/30(木) 23:26:03 ID:yVhW4Ory.net
とりあえずt = 1/(n-1)とおいてt=0の近傍では左辺-右辺は
4 t^2 + (13 t^3)/3 + (11 t^4)/3 + (8 t^5)/5 - (7 t^6)/90 + O(t^7)
(テイラー級数)
だそうな

714 :132人目の素数さん:2021/10/01(金) 06:04:21 ID:y+GdRVMF.net
初項は 2n/(n-1)^2 > 2/(n-1),
あとの項も >0,

なお、二項公式から
 Σ(k=1〜2n) C(2n,k) (t^2)^k = (1+t^2)^{2n} - 1.

715 :132人目の素数さん:2021/10/18(月) 20:54:00 ID:gvNZ1Lh7.net
0<a<b,A=(a+b)/2 , G=(ab)^(1/2), H=2ab/(a+b), I(a,b)≡a(a+3b)/(3a+b) とおくと
H≦I(a,b)≦G≦I(b.a)≦A 
が成立するんですけどI(a,b)の意味というか簡単な解釈みたいのってあるのでしょうか?
証明はただ計算すればいいんですけど、、

716 :132人目の素数さん:2021/10/19(火) 18:46:50 ID:OvSIJGC7.net
分かりませんね。

 I(a,b) = (A+b)/(1+b/H),
 I(b,a) = (A+a)/(1+a/H),
なので
 f(x) = (A+x)/(1+x/H),
とおいてみますか。
これはxの一次分数式で単調減少です。
 f(∞) = H,
 f(b) = I(a,b),
 f(H) = (A+H)/2,
 f(G) = √(AH) = G,
 f(A) = 2AH/(A+H),
 f(a) = I(b,a),
 f(0) = A,

717 :132人目の素数さん:2021/10/19(火) 19:59:53 ID:G5c9BaN8.net
>>715

AとHの調和平均を J(a,b)=2AH/(A+H)=4ab(a+b)/(a^2+b^2+6ab) とすると

H≦I(a,b)≦J(a,b)≦G≦ab/J(a,b)≦I(b,a)≦A も成立

718 :132人目の素数さん:2021/10/20(水) 04:41:32 ID:tGBp8wkO.net
>>716
f(A) = 2AH/(A+H) = J(a,b),
f(H) = (A+H)/2 = ab/J(a,b),

719 :132人目の素数さん:2021/10/20(水) 10:16:30 ID:Xr1EP2wS.net
f(-a)=b
f(-b)=a

720 :132人目の素数さん:2021/10/20(水) 19:32:20 ID:Xr1EP2wS.net
 f(x) = (A+x)/(1+x/H) ⇔ (f-H):(A-f)=H:x (H,Aの内分比をxで定める)
 g(x) = (b+x)/(1+x/a) ⇔ (g-a):(b-g)=a:x (a,bの内分比をxで定める)

 f(-A) = 0        g(-b) = 0
 f(-b) = a        g(∞) = a
 f(∞) = H        g(b) = H
 f(b) = I(a,b)      g(A) = I(a,b)
 f(A) = 2AH/(A+H)  g(I(b,a)) = 2AH/(A+H)
 f(G) = √(AH) = G  g(G) = √(AH) = G
 f(H) = (A+H)/2    g(I(a,b)) =(A+H)/2
 f(a) = I(b,a)      g(H) = I(b,a)
 f(0) = A        g(a) = A
 f(-a) = b        g(0) = b
 f(-H) = ∞       g(-a) =∞

721 :132人目の素数さん:2021/10/24(日) 18:45:19 ID:P60+HIiU.net
久々に来たまだあったんだ
aopsは計算機でSOSするのが多く手だと難しいのが多いね
あとは巡回式とか、巡回式じゃない非対称とかも最近多い
不等式を解くテクニックとかは飽和かな?

722 :132人目の素数さん:2021/10/26(火) 14:23:08 ID:yHWX4IjJ.net
AoPS = Art of Problem Solving だろうな。
http://artofproblemsolving.com/

723 :132人目の素数さん:2021/11/02(火) 01:18:14 ID:+0fDU6q/.net
z∈C、|z|≦1のとき、|(3z-1)/(z-3)| の最大値を求めよ。

724 :132人目の素数さん:2021/11/02(火) 01:23:12 ID:M1RfnTE1.net
閉円盤

725 :132人目の素数さん:2021/11/02(火) 05:33:41 ID:te4HpQwE.net
解1.
A(1/3,0) B(3,0) P(x,y) とおくと
 (与式) = 3(AP/BP)

(与式) = k (<3) は
 AP:BP = k:3
なる アポロニウスの円である。
直径の両端は 内分点と外分点
 (3-8/(3-k),0) と (3-8/(3+k),0)
閉円盤 xx+yy≦1 と共有点をもつことから
 3-8/(3-k) ≧ -1 または 3-8/(3+k) ≦ 1,
∴ k ≦ 1,

解2.
|z-3|^2 - |3z-1|^2
 = (z-3)(z~-3) - (3z-1)(3z~-1)
 = 8(1-zz~)
 = 8(1-|z|^2),
あるいは
|r・e^(iθ)-3|^2 - |3r・e^(iθ)-1|^2
 = (rr -6r・cosθ +9) - (9rr - 6r・cosθ +1)
 = 8(1-rr),
よって
 |z|≦1  ⇔  |(3z-1)/(z-3)| ≦ 1,

726 :132人目の素数さん:2021/11/02(火) 06:43:06 ID:te4HpQwE.net
〔補題〕
d(X) がある空間における距離ならば
 D(X) = (1/2)log{h + d(X)^2},   h≧4/3
も距離。

[面白スレ39.399,402,404]

727 :132人目の素数さん:2021/11/02(火) 19:37:00 ID:te4HpQwE.net
は怪しいので
 D(X) = log{1 + d(X)},
に訂正…

728 :132人目の素数さん:2021/11/04(木) 22:55:14 ID:n3XtJQuq.net
>>723の類題。
z∈C、|z|≦1のとき、|z(z-1)(z-2)| の最大値を求めよ。

729 :132人目の素数さん:2021/11/04(木) 23:33:02 ID:+6XnN/it.net
z=-1で最大

730 :132人目の素数さん:2021/11/06(土) 15:34:08 ID:QOJe0Sk2.net
 |z| ≦ 1,
 |z-1| ≦ |z| + 1 ≦ 2,
 |z-2| ≦ |z| + 2 ≦ 3,
 ……
辺々掛けて
 |z(z-1)(z-2)…(z-n)| ≦ (n+1)!,
等号は z=-1 のとき。

731 :132人目の素数さん:2021/11/06(土) 17:32:30 ID:WFsWEsWR.net
なんか凄いな

732 :132人目の素数さん:2021/11/15(月) 07:22:04 ID:0+gULGFn.net
z≦1(円盤)程度ならプロットすればいいんよ(思考放棄)

733 :132人目の素数さん:2021/11/15(月) 18:53:19 ID:EETFS5Z+.net
凄くはないよ
その設定で作問してる
等号成立条件が一致してるとか出来過ぎ
汎用性がない解法

734 :132人目の素数さん:2021/11/20(土) 01:53:37 ID:ecvBNjJu.net
〔問題471〕
任意の自然数nに対して、次を示せ。
 1/n > 1/(n+1) + 1/(n+1)(n+2) + 1/(n+1)(n+2)(n+3) + ……

[高校数学の質問スレPart414.471]

735 :132人目の素数さん:2021/11/21(日) 02:46:04 ID:myOhL9Wf.net
右辺は ∫[0,1] x^n e^{1-x} dx になるらしい…

[同.492]

736 :132人目の素数さん:2021/11/21(日) 13:18:27 ID:z//mcKXR.net
むむむ…

737 :132人目の素数さん:2021/11/21(日) 20:28:37 ID:myOhL9Wf.net
右辺は {n!e} になるらしい。

高校生なら等比級数
 1/n = 1/(n+1) + 1/(n+1)^2 + 1/(n+1)^3 + ……
を使うだろうけど...

738 :132人目の素数さん:2021/11/21(日) 21:24:18 ID:FQCqRacp.net
>>737
そのまんまじゃないか。全く気付かずガックリ。そして>>735となるのも知ってる人は知っているヤツよなあ(積分がある値になることを知っていれば、勘で逆算できる可能性がある)

739 :132人目の素数さん:2021/11/22(月) 05:31:37 ID:ezmXDy6Q.net
>>735
 被積分関数は x≒1 で急増するから、そこで精度が必要。
しかし x=1 で上から接するのは、e^{1-x} が下に凸なので難しい。
そこで e^{x-1} ≧ x と下から接すれば
 e^{1-x} ≦ 1/x,
∴ (与式) < ∫[0,1] x^{n-1} dx = 1/n を得る。

740 :132人目の素数さん:2021/11/24(水) 06:01:42 ID:JOGGpS/y.net
〔補題292〕
a,b,c,d > 0 のとき
 a/b + b/c + c/d + d/a ≧ 8(ac+bd)/((a+c)(b+d)),
等号成立は a=c, b=d のとき。

[分かスレ471.279,292]

741 :132人目の素数さん:2021/11/24(水) 23:12:24 ID:4mZSj8jn.net
>>740

a/b + b/c + c/d + d/a
≧ 2√{(ac)/(bd)}+ 2√{(bd)/(ac)}
= 2(ac + bd) / √(abcd)
≧ 8(ac + bd) / {(a + c)(b + d)}

742 :132人目の素数さん:2021/11/25(木) 07:10:29 ID:UgfLf66S.net
正解です!!
AM-GM だけでいけますね。

743 :132人目の素数さん:2021/11/27(土) 12:53:00 ID:HxEDg/nu.net
〔掛谷の定理〕
 a_n z^n - a_(n-1) z^(n-1) + …… + (-1)^(n-1) a_1 z + (-1)^n a_0 = 0 (a_k>0)
の根は m ≦ |z| ≦ M をみたす。
ここに m = min{a0/a1, a1/a2, …, a(n-1)/a_n}
   M = Max{a1/a0, a2/a1, …, a(n-1)/a_n}

高橋正明 著「複素数」科学新興社モノグラフ13. (1972)
高橋正明 著「複素数」改訂版, 科学新興新社モノグラフ9. (1998)

すべて実根のときは、ニュートンの不等式から
  a(k-1)/a_k ≦ a_k/a(k+1),
 m = a_0/a_1 ≦ …… ≦ a(n-1)/a_n = M,

∴ a_0/a_1 ≦ z ≦ a(n-1)/a_n,

744 :132人目の素数さん:2021/11/27(土) 14:13:07 ID:T4OPSuH4.net
>>741
ナイスエレガンス

745 :132人目の素数さん:2021/11/28(日) 11:20:03 ID:MWTbmNPN.net
>>743
(訂正スマソ)
  M = Max{a0/a1, a1/a2, …, a(n-1)/a_n}

元の形は
 0 < a_0 < a_1 < … < a(n-1) < a_n のとき |z| <1,
 a_0 > a_1 > … > a(n-1) > a_n > 0 のとき |z| >1,

746 :132人目の素数さん:2021/12/19(日) 06:22:06 ID:N+EeFsux.net
2018年度奈良県立医科大学後期四番

https://i.imgur.com/3OlENLH.png

747 :132人目の素数さん:2021/12/20(月) 09:14:39 ID:d28R2ON3.net
>>746
これは大学の解析の問題やな
普通の高校生には手も足も出ないだろな
東大の問題よりセンスがあってカッコ良いわ

748 :132人目の素数さん:2021/12/20(月) 09:45:52 ID:HFnFoVGH.net
相変わらず手抜きだよ

749 :132人目の素数さん:2021/12/20(月) 11:21:02 ID:NKcN+ZAk.net
全然手抜きじゃないだろ
手抜きというのは東大の円周率の評価式や加法定理の証明問題をいうんだよ

750 :132人目の素数さん:2021/12/20(月) 14:52:43 ID:HFnFoVGH.net
数学できるヤツいらんから、大学で扱ってる問題拾ってきて出してるだけ。

東大のは採点が大変なんだから手抜きにはならん。(想定解答をもとに採点するが別解が出てくれればまた採点官で検討して反映させるので)

751 :132人目の素数さん:2021/12/20(月) 15:56:26 ID:d28R2ON3.net
別解が出てくるのはどんな問題でも同じだろ

752 :132人目の素数さん:2022/01/07(金) 20:39:08 ID:uuJtvVgV.net
a,b,c>0,
ab+bc+ca=12,
√(ab) + √(bc) + √(ca) + 32/(abc) ≧10.

( ゚∀゚) プケラッチョ

753 :132人目の素数さん:2022/01/11(火) 19:08:21 ID:V9PX5rXA.net
任意の実数xで、cos(cosx) > sin(sinx)

754 :132人目の素数さん:2022/01/11(火) 20:20:46 ID:V9PX5rXA.net
>>752
条件式の相加相乗から8≧abcなので
√(ab)+√(bc)+√(ca)+16/(abc)+16/(abc)≧5(256/(abc))^(1/5)≧10

755 :132人目の素数さん:2022/01/28(金) 22:49:52 ID:FY6nRJtW.net
1997東大理系数学第2問

756 :132人目の素数さん:2022/02/06(日) 19:19:12 ID:LRt8HG3c.net
z∈C、|z|=1 に対して、
|e^z - z^3 - z - i| ≦ e - 1.

757 :132人目の素数さん:2022/03/12(土) 20:51:17 ID:+WWCnbfX.net
複素関数の本を読んでいたら、ハルナックの不等式が出てきた。春泣く不等式

758 :132人目の素数さん:2022/04/13(水) 20:44:36.01 ID:on0g0jTO.net
z∈C に対して |z(4-z)|<1 を解きたいんですけど、どうやればいいんでせうか?

759 :132人目の素数さん:2022/04/13(水) 21:33:37.94 ID:cxjLeouM.net
ときたいとは?
面積求めるとか?
図示するとか?

760 :132人目の素数さん:2022/04/14(木) 00:03:20.04 ID:RdL/Dtg3.net
図示ですぞ

761 :132人目の素数さん:2022/04/14(木) 00:14:30.93 ID:UD9sHCgS.net
>>758
>>723-730らへん
20年前に高校でやったなあ

762 :132人目の素数さん:2022/04/14(木) 00:17:50.63 ID:UD9sHCgS.net
ごめん違う問題だったな。平方完成しててきとうにやればええやろ。

763 :132人目の素数さん:2022/04/14(木) 00:28:38.03 ID:VImhL7xV.net
大先生に書いてもらうとこんな感じ
https://www.wolframalpha.com/input?i=%7C%28x%5E2%2By%5E2%29%28%284-x%29%5E2%2By%5E2%29%7C%3D1&lang=ja

764 :132人目の素数さん:2022/04/14(木) 00:57:08.49 ID:RdL/Dtg3.net
大先生は よく分からんね。
2つの楕円の内部が解領域なのか…

765 :132人目の素数さん:2022/04/14(木) 01:09:54.51 ID:wrO3la+m.net
>>764
最後の方にある

y = sqrt(-x^2 + sqrt(16 x^2 - 64 x + 65) + 4 x - 8)



y = -sqrt(-x^2 + sqrt(16 x^2 - 64 x + 65) + 4 x - 8)

の間やろ

766 :132人目の素数さん:2022/04/14(木) 02:27:28 ID:UD9sHCgS.net
>>764
スケールおかしいだけで、円の内部やぞ。
|z(4-z)|=|z||z-4|だから原点0とzのキョリと4とzのキョリの積、それが1未満。0付近か4付近だと小さくなる。

767 :132人目の素数さん:2022/04/14(木) 03:05:03.21 ID:xiGeSXG1.net
>>766大先生の答え整理したら円になる?

768 :132人目の素数さん:2022/04/14(木) 04:05:38 ID:RdL/Dtg3.net
zの存在領域は、0の近傍と4の近傍に2つある。
境界線は次式で表されて、楕円っぽい形で、円にはならない。
y=-sqrt{-xx+4x-8+√(16xx-64x+65)}
y= sqrt{-xx+4x-8+√(16xx-64x+65)}.

769 :132人目の素数さん:2022/04/14(木) 12:44:16.36 ID:l1Bgx+ly.net
ごめんボーッとしてて頭おかしくなってたわ。
因数分解して(二次式)(二次式)<0にできるんだから楕円2つの内部だ。楕円のようなカタチの二次式は楕円しかないので。

>>766
>>764
一行目完全にムシ、それ以外は正しい。

770 :132人目の素数さん:2022/04/14(木) 13:42:00.94 ID:4XM7DyMd.net
イヤいくらでもあるやろ
y^2 = - (x^2-1)*(x^2-3) (楕円曲線(楕円二つではない))
とか
https://www.wolframalpha.com/input?i=y%5E2+%3D+-+%28x%5E2-1%29*%28x%5E2-3%29&lang=ja

771 :132人目の素数さん:2022/04/14(木) 18:34:50.58 ID:RdL/Dtg3.net
>>769
正確にはだえんじゃないよね。
y=-sqrt{-xx+4x-8+√(16xx-64x+65)}
で言うと、左半分が少し広がっている。

772 :132人目の素数さん:2022/04/14(木) 18:47:04.72 ID:RdL/Dtg3.net
もともとは、冪級数
 S(z) = Σ[n=0 to ∞] {z(4-z)}^n
について、
(1) S(z) の収束する領域が2つの分離した領域であることを示せ。
(2) S(z) を最大限に解析接続せよ。
という問題。

 y=-sqrt{-xx+4x-8+√(16xx-64x+65)}
 y= sqrt{-xx+4x-8+√(16xx-64x+65)}.

これって微分して増減表調べればグラフの概形が分かるだろうけどダルイ。

(1)を、z = re^(iθ)、4-z = se^{ i (φ+)π} とおいて、極形式で考える方法は挫折。

773 :132人目の素数さん:2022/04/14(木) 20:37:32.40 ID:UD9sHCgS.net
>>770
x・yの二次式って意味だった。

>>771
ホンマや、図示と違って楕円じゃないな。。。ちゃんとした図示じゃなかったってだけか。
てっきり因数分解=0とできるもんだと誤解したわ。

774 :132人目の素数さん:2022/04/15(金) 13:42:40.32 ID:t03sEjFl.net
youtube,twitterの不等式botさんは望月教授の弟子?で女性?

775 :132人目の素数さん:2022/04/15(金) 21:54:29.68 ID:KFvGzJHP.net
Σ_{i=1}^n Σ_{j=1}^n |x_i-x_j|^(1/2)≦Σ_{i=1}^n Σ_{j=1}^n |x_i+x_j|^(1/2)

776 :132人目の素数さん:2022/04/17(日) 19:45:28.64 ID:p0ajF9zH.net
x,y,zが非負実数のとき Σ(x+y)^(-2)≧9/(4Σxy)

777 :132人目の素数さん:2022/04/18(月) 08:19:06.84 ID:4YPe9yxz.net
a,b,c,d > 0 のとき
1/(1/a + 1/b) + 1/(1/c + 1/d) ≦ 1/{1/(a+c) + 1/(b+d)}.

778 :132人目の素数さん:2022/04/19(火) 04:42:19.83 ID:pmf8vt90.net
USAMO第六問

nは2以上の整数
実数列x_1≧x_2≧…≧x_n,y_1≧y_2≧…≧y_n があり、次式を満たしている
・0=Σx_i=Σy_i
・1=Σ(x_i^2)=Σ(y_i^2)
このとき
Σ(x_i*y_i-x_i*y_(n+i-1))≧2/√(n-1)
を示せ

779 :132人目の素数さん:2022/04/19(火) 04:43:45.43 ID:pmf8vt90.net
>>778
2020年度のUSAMO
https://artofproblemsolving.com/community/c1209089_2020_usomo

780 :132人目の素数さん:2022/04/19(火) 12:55:48.55 ID:hd+PG5o3.net
>>777
(a+c)(b+d)-(a+b+c+d)(ab/(a+b)+cd/(c+d))=(ad-bc)^2/(a+b)(c+d)≧0

781 :132人目の素数さん:2022/04/25(月) 02:00:10.86 ID:J6JlHhd4.net
ツイッターで拾った謎定数
https://i.imgur.com/UspzFSB.png

782 :132人目の素数さん:2022/04/25(月) 17:46:55.78 ID:EdwWMLBZ.net
これな
https://i.imgur.com/jg0aj9g.jpg
シンプルに見える問題でも複雑な数になる好例

783 :132人目の素数さん:2022/04/25(月) 17:53:23.58 ID:EdwWMLBZ.net
同じくシンプルだが複雑な定数になる例、
あやな(@suugaku1)の不等式
-M≦sinx+sin2x≦M
のMも偶然だが1.76くらいになる

784 :132人目の素数さん:2022/04/25(月) 20:00:08.42 ID:q7yGPC6B.net
ウクライナMO2021

a≧b≧c>0のとき
(a^2+b^2)/(a+b)+(b^2+c^2)/(b+c)+(c^2+a^2)/(c+a)≧a+b+c+(a-c)^2/(a+b+c)

785 :132人目の素数さん:2022/04/26(火) 00:15:41.36 ID:zSKeHW9H.net
>>782
どうやって導き出したんですか?

786 :132人目の素数さん:2022/04/26(火) 00:34:34.71 ID:NvtQufOW.net
代数関数の値域の上限、下限は係数体の代数的拡大の元
決定するためのアルゴリズムも見つかってる

787 :132人目の素数さん:2022/04/27(水) 21:17:46.91 ID:3pZmYP1d.net
>>784 abcの大小は関係ない

788 :132人目の素数さん:2022/04/28(木) 00:41:06.46 ID:sAOcG1R4.net
コンテストで出題されたアシンメトリーな不等式をもっとくれ

789 :132人目の素数さん:2022/05/01(日) 13:38:24.60 ID:69yOeg50.net
>>781 >>782
nを4から正の偶数に一般化してみた

任意の相異なる実数a,b,cに対し
(a/(b-c))^n + (b/(c-a))^n + (c/(a-b))^n ≧ (2-k)/(2k-k^2)^(n/2)
ここにkは -1+x+2x^n = 0 の正の根

n≧4で等号成立は以下の6組
a:b:c = k±√(2k-k^2) : k±k√(2k-k^2) : -k(1-k)
a:b:c = k±k√(2k-k^2) : k±√(2k-k^2) : -k(1-k)
a:b:c = k±√(2k-k^2) : -(-k±√(2k-k^2)) : 2

790 :132人目の素数さん:2022/05/05(木) 15:25:37.46 ID:R+LgARSS.net
x,y,z>0⇒4Σ((x+y)(y+z))^2≧Σxy(3x+3y+2z)^2

791 :132人目の素数さん:2022/05/25(水) 22:54:43.19 ID:nNeYhYVz.net
はい
https://i.imgur.com/PGGXthW.png

792 :132人目の素数さん:2022/07/20(水) 02:52:11.46 ID:83qPeNBR.net
w∈C、|w| < 1/2 に対して、|e^w -1| < |w|e^{1/2}.

w∈Rのときは分かるけど、複素数のときはどうやって証明するんでしょうか?

793 :132人目の素数さん:2022/07/20(水) 08:22:42 ID:d+g2kWua.net
f(z) = eᶻ-1とおけば最大値の原理から|z|<1/2に対して

|eᶻ-1|/|z| < max { |eᶻ-1|/z ; |z| = 1/2 }
= 2((e^(1/2)-1) (z = 1/2のとき最大値)
= 1.2974425414
≦ e^(1/2) = 1.6487212707

794 :132人目の素数さん:2022/07/20(水) 13:25:18.94 ID:rsT8xIwC.net
>>793
なるほど、ありがとうございます。

795 :132人目の素数さん:2022/07/20(水) 13:45:48 ID:rsT8xIwC.net
>>793
ごめん、やっぱ分かってなかった。
境界 |z| = 1/2 で最大値をとることまでは分かったけど、z=1/2で最大となるのはなぜですか?

|z| = 1/2のときに、|(e^z -1)/z| = 2|e^z -1| までは分かるけど、
2|e^z -1| ≦ 2(e^{1/2} -1) はなぜ?

796 :132人目の素数さん:2022/07/20(水) 15:00:00.10 ID:kuOeuKMZ.net
|e^w-1|
=|w^1/1!+w^2/2!+w^3/3!+...|
<=|w|(1/1!+|w|/2!+|w|^2/3!+...)
<=|w|(1/1!+(1/2)/2!+(1/2)^2/3!+...)
=|w|(e^(1/2)-1)/(1/2).

797 :132人目の素数さん:2022/07/20(水) 17:09:23.51 ID:rsT8xIwC.net
>>796
なるほど理解できました。ありがとうございまする。

798 :132人目の素数さん:2022/08/04(木) 02:20:43.56 ID:2zMvcFob.net
単位円上の5点を取り10本の線分で結ぶと凸包の五角形が、11個の部分に分割される
中央の小五角形の面積をT、小五角形と辺を共有する5つの三角形の面積の総和をSとする
S + 2Tの最大値を求めよ

799 :132人目の素数さん:2022/08/17(水) 17:06:39.02 ID:VYryJHFS.net
>>756
これは どうやって証明するのでしょうか?

800 :132人目の素数さん:2022/08/17(水) 20:48:07.18 ID:x5NeuThT.net
大先生いわく成立してないぽい

https://www.wolframalpha.com/input?i=abs%28+exp%28exp%28i+x%29%29+-+exp%28i+x%29+-+exp%283i+x%29+-+i%29&lang=ja

801 :132人目の素数さん:2022/08/17(水) 20:57:45.28 ID:VYryJHFS.net
>>800
全く違う式では?

802 :132人目の素数さん:2022/08/17(水) 21:07:15.27 ID:x5NeuThT.net
何故?

803 :132人目の素数さん:2022/08/17(水) 21:13:20.13 ID:VYryJHFS.net
>>802
ああ、置き換えていたんですな。ごめそ。

804 :132人目の素数さん:2022/08/17(水) 21:14:25.41 ID:VYryJHFS.net
右辺は e+1 でしょうね。

805 :132人目の素数さん:2022/08/17(水) 22:01:18.98 ID:VYryJHFS.net
>>756
改造してみた。どう?

z∈C、|z|=1 に対して、
|e^z - z^3 - z - i| ≦ e + (√2) - 5/3.

806 :132人目の素数さん:2022/08/17(水) 22:33:15.76 ID:j8RRYzIG.net
5/3? 4/3でなく?

807 :132人目の素数さん:2022/08/17(水) 23:01:03.95 ID:j8RRYzIG.net
やっぱりおかしいやろ
大先生によるとx=2.987のとき2.72921になる

https://www.wolframalpha.com/input?i=abs%28+exp%28exp%28i+x%29%29+-+exp%28i+x%29+-+exp%283i+x%29+-+i%29+when+x+%3D+2.987&lang=ja

e+√(2)-5÷3
=2.465828724165
では抑えられん

808 :132人目の素数さん:2022/08/17(水) 23:29:50.23 ID:VYryJHFS.net
>>807
|z|=1なのに?

809 :132人目の素数さん:2022/08/17(水) 23:31:17.46 ID:VYryJHFS.net
すまん。

810 :132人目の素数さん:2022/08/17(水) 23:57:56.28 ID:VYryJHFS.net
>>806
計算しなおしたら、4/3でした。 すまそ。

z∈C、|z|=1 に対して、
|e^z - z^3 - z - i| ≦ e + (√2) - 4/3.

811 :132人目の素数さん:2022/09/06(火) 19:26:53.16 ID:na6+6u4X.net
3sinθ/(2+cosθ) < θ < (2sinθ+tanθ)/3

812 :132人目の素数さん:2022/09/07(水) 01:42:03.86 ID:fi9kzCby.net
>>811
スネル・ホイヘンスの不等式
>>634-650

813 :132人目の素数さん:2022/09/09(金) 10:08:28.12 ID:BXbMv5wz.net
ほー、スネルとホイヘンスといえば光学か
確かに光学(の何かに)使えそうな見た目してんな>>812

814 :132人目の素数さん:[ここ壊れてます] .net
十分小さな z∈C に対して、|z|/2 ≦ |log(1+z)| ≦2|z|.
これはどうやって証明するのでせうか?

815 :132人目の素数さん:[ここ壊れてます] .net
マクローリン展開じゃないの?
| log(1+z) - z | ≦ Σ[k≧2] |zᵏ|/k
≦ Σ[k≧2] |zᵏ|/2
= |z|²/(1-|z|)

816 :132人目の素数さん:2022/09/10(土) 06:57:25.66 ID:IWIXvTBm.net
う~ん

817 :132人目の素数さん:2022/09/12(月) 17:26:45.33 ID:moLjFVx4.net
理解した。
|Log(1+z)|/|z| = 1 + Σ[k≧2] (1/k)*|z|^{k-1} → 1 (|z|→0)

818 :132人目の素数さん:2022/09/14(水) 17:16:59.13 ID:zemgK99G.net
>>814
改造してみた。うひょっ!

|z|< 1/2、z∈C に対して、|z|/2 ≦ |log(1+z)| ≦ 3|z|/2..

819 :132人目の素数さん:[ここ壊れてます] .net
a,b,c,d >0 に対して
(ab)^(1/3) + (cd)^(1/3) ≦ {(a+b+c)(a+c+d)}^(1/3).

820 :132人目の素数さん:2022/09/26(月) 18:16:13.36 ID:wmq00M3D.net
z∈C に対して、|e^z - 1 - z| ≦ (3/4)|z|.

821 :132人目の素数さん:2022/09/26(月) 20:39:38.62 ID:FSZDCr7O.net
>>820
|z| \leq 1が必要じゃの。

822 :132人目の素数さん:2022/09/27(火) 00:51:21.62 ID:3dnqmVEF.net
| (exp(z)-1-z)/z |
= | Σ[ k≧2 ] zᵏ⁻¹/k! |
≦ | Σ[ k≧2 ] | z Iᵏ⁻¹/k!
≦ | Σ[ k≧2 ] 1/k!
= exp(1) -2
≦ 1/2+1/6(1/(1-1/4))
= 13/18
< 3/4

823 :132人目の素数さん:2022/12/20(火) 15:42:24.25 ID:R0GrT6qP.net
https://i.imgur.com/bFfDS2f.jpg
https://i.imgur.com/JVQsjAd.jpg
https://i.imgur.com/I1aT8Pu.jpg
https://i.imgur.com/doMWwIW.jpg
https://i.imgur.com/8cAJeMD.jpg
https://i.imgur.com/crize0k.jpg
https://i.imgur.com/o7IYRlH.jpg
https://i.imgur.com/jkYopCU.jpg
https://i.imgur.com/4aRCLH9.jpg
https://i.imgur.com/HkEhMPT.jpg
https://i.imgur.com/yjOkcCb.jpg
https://i.imgur.com/KQeyoRY.jpg

824 :132人目の素数さん:2023/02/28(火) 18:31:31.57 ID:gK2nVmwr.net
https://i.imgur.com/lEXzgmV.jpg

825 :132人目の素数さん:2023/03/03(金) 04:38:08.86 ID:bTCAa9Qt.net
ちらっと立ち読みしただけなんだが、数蝉のNoteに、階乗のAM-GMが載っていたよ、

826 :132人目の素数さん:2023/05/21(日) 10:43:46.51 ID:t4b79528.net
π: R→R/I

827 :132人目の素数さん:2023/05/21(日) 10:45:48.42 ID:t4b79528.net
π: a→a'

828 :132人目の素数さん:2023/05/21(日) 10:46:23.12 ID:t4b79528.net
自然な準同型写像

829 :132人目の素数さん:2023/05/21(日) 10:48:51.65 ID:t4b79528.net
Kerf=f⁻¹(0)はIdeal
f(x)=0のx原像

830 :132人目の素数さん:2023/05/21(日) 10:56:30.65 ID:t4b79528.net
準同型写像fの核Ker

831 :132人目の素数さん:2023/05/21(日) 10:57:32.36 ID:t4b79528.net
Kerf=(0)⇔fは単射

832 :132人目の素数さん:2023/05/21(日) 10:59:21.23 ID:t4b79528.net
R/Kerf≅f(R)

833 :132人目の素数さん:2023/05/21(日) 11:01:37.44 ID:t4b79528.net
f'は単準同型写像
剰余環→環

834 :132人目の素数さん:2023/05/21(日) 11:02:21.15 ID:t4b79528.net
f: R→R'

835 :132人目の素数さん:2023/05/21(日) 11:02:52.23 ID:t4b79528.net
π: R→R/I、I=Kerf

836 :132人目の素数さん:2023/05/21(日) 11:03:42.70 ID:t4b79528.net
f': R/I→R'

837 :132人目の素数さん:2023/05/21(日) 11:13:01.66 ID:t4b79528.net
R上の不定元、変数

838 :132人目の素数さん:2023/05/21(日) 11:13:59.15 ID:t4b79528.net
R上のXの多項式

839 :132人目の素数さん:2023/05/21(日) 12:08:26.23 ID:LagPnPNw.net
exp(x)>x

840 :132人目の素数さん:2023/05/21(日) 12:18:01.48 ID:bhyiMb+P.net
次数n=degf(x)
-∞

841 :132人目の素数さん:2023/05/21(日) 12:46:41.12 ID:p6AFiZvO.net
R[x]

842 :132人目の素数さん:2023/05/21(日) 12:51:52.26 ID:p6AFiZvO.net
f+g=∑(a+b)x

843 :132人目の素数さん:2023/05/21(日) 12:54:49.61 ID:p6AFiZvO.net
fg=∑∑(ab)x

aᵢbⱼxᵏ (i+j=k)
k=0~n

844 :132人目の素数さん:2023/05/21(日) 12:55:44.38 ID:p6AFiZvO.net
R上の1変数多項式環

845 :132人目の素数さん:2023/05/21(日) 13:03:03.60 ID:p6AFiZvO.net
a₀=0、0元
a₀=1、単位元

846 :132人目の素数さん:2023/05/21(日) 13:10:03.75 ID:p6AFiZvO.net
degfg=degf+degg
最高次の係数≠0
aᵢ、bⱼ
aᵢbⱼxⁱ⁺j
a=0またはb=0⇒-∞

847 :132人目の素数さん:2023/05/21(日) 13:11:17.11 ID:p6AFiZvO.net
Rが整域⇒R[X]は整域

848 :132人目の素数さん:2023/05/21(日) 13:12:44.87 ID:p6AFiZvO.net
R[X][Y]=R[X, Y]

849 :132人目の素数さん:2023/05/22(月) 03:18:58.93 ID:2UAex/JZ.net
最大公約元

850 :132人目の素数さん:2023/05/22(月) 03:19:56.92 ID:2UAex/JZ.net
原始多項式

851 :132人目の素数さん:2023/05/22(月) 03:20:38.92 ID:2UAex/JZ.net
係数の最大公約元=1の時,
原始多項式

852 :132人目の素数さん:2023/05/22(月) 03:21:39.91 ID:2UAex/JZ.net
Gaussの補題

853 :132人目の素数さん:2023/05/22(月) 03:22:23.51 ID:2UAex/JZ.net
f, gが原始多項式⇒積fgは原始多項式

854 :132人目の素数さん:2023/05/22(月) 03:23:54.64 ID:2UAex/JZ.net
f=uF

855 :132人目の素数さん:2023/05/22(月) 03:37:03.22 ID:2UAex/JZ.net
原始多項式Fは可逆元の積を除いて一意的

856 :132人目の素数さん:2023/05/22(月) 03:37:49.60 ID:2UAex/JZ.net
UFD、Rの商体をKとする

857 :132人目の素数さん:2023/05/22(月) 03:38:44.86 ID:2UAex/JZ.net
Fを原始多項式

858 :132人目の素数さん:2023/05/22(月) 03:39:16.44 ID:2UAex/JZ.net
しなわち(a₁)=1とする

859 :132人目の素数さん:2023/05/22(月) 03:41:27.62 ID:2UAex/JZ.net
ある原始多項式Fに対して
f=uFと表せる
uは可逆元、単元

860 :132人目の素数さん:2023/05/22(月) 03:44:38.96 ID:2UAex/JZ.net
Eisensteinの既約性判定条件

861 :132人目の素数さん:2023/05/22(月) 03:47:12.97 ID:2UAex/JZ.net
fがKの素元⇒FはRの素元

862 :132人目の素数さん:2023/05/22(月) 03:48:20.76 ID:2UAex/JZ.net
特にF自身をとると

863 :132人目の素数さん:2023/05/22(月) 03:49:36.72 ID:2UAex/JZ.net
特にFがKで素元であれば

864 :132人目の素数さん:2023/05/22(月) 03:51:04.42 ID:2UAex/JZ.net
FはKで素元であればRで素元である

865 :132人目の素数さん:2023/05/22(月) 03:51:59.04 ID:2UAex/JZ.net
RがUFD⇒R[はUFD]

866 :132人目の素数さん:2023/05/22(月) 03:54:39.98 ID:2UAex/JZ.net
特にKが体⇒Kは整域⇒K[x]はUFD

867 :132人目の素数さん:2023/05/22(月) 03:55:33.97 ID:2UAex/JZ.net
RをUFD、

868 :132人目の素数さん:2023/05/22(月) 03:55:54.15 ID:2UAex/JZ.net
Kをその商体とする。

869 :132人目の素数さん:2023/05/22(月) 03:59:08.23 ID:2UAex/JZ.net
p∤aₙ∧p|その他∧p²∤a₀
pは素元⇒f(x)は既約多項式

870 :132人目の素数さん:2023/05/22(月) 04:00:30.30 ID:2UAex/JZ.net
モニックp∤aₙ
p|その他0~n-1
p²∤a₀、定数項

871 :132人目の素数さん:2023/05/22(月) 04:01:37.29 ID:2UAex/JZ.net
Eisensteinの既約性判定法

872 :132人目の素数さん:2023/05/22(月) 04:02:37.04 ID:2UAex/JZ.net
素元pで
0、1、1、…、1、最後は≠2

873 :132人目の素数さん:2023/05/22(月) 04:27:14.46 ID:2UAex/JZ.net
有限体

874 :132人目の素数さん:2023/05/22(月) 04:27:56.66 ID:2UAex/JZ.net
Kを有限体とする

875 :132人目の素数さん:2023/05/22(月) 04:28:27.35 ID:2UAex/JZ.net
Kの単位元を1とする

876 :132人目の素数さん:2023/05/22(月) 04:29:05.31 ID:2UAex/JZ.net
(n)=n・1

877 :132人目の素数さん:2023/05/22(月) 04:29:56.17 ID:2UAex/JZ.net
準同型写像τ: ℤ→K

878 :132人目の素数さん:2023/05/22(月) 04:30:41.98 ID:2UAex/JZ.net
環準同型定理より

879 :132人目の素数さん:2023/05/22(月) 04:31:13.14 ID:2UAex/JZ.net
π: ℤ→ℤ/Kerτ

880 :132人目の素数さん:2023/05/22(月) 04:31:45.99 ID:2UAex/JZ.net
τ': ℤ/Kerτ→K

881 :132人目の素数さん:2023/05/22(月) 04:32:50.30 ID:2UAex/JZ.net
同型
ℤ/Kerτ≅τ(ℤ)=K

882 :132人目の素数さん:2023/05/22(月) 04:33:57.73 ID:2UAex/JZ.net
τ(ℤ)は体Kの部分環だから整域である

883 :132人目の素数さん:2023/05/22(月) 04:34:37.36 ID:2UAex/JZ.net
Kerτはℤの素Idealとなる

884 :132人目の素数さん:2023/05/22(月) 04:35:54.91 ID:2UAex/JZ.net
Kerτ=(p)

885 :132人目の素数さん:2023/05/22(月) 04:36:41.78 ID:2UAex/JZ.net
体Kの標数p

886 :132人目の素数さん:2023/05/22(月) 04:37:32.28 ID:2UAex/JZ.net
これはKの素体

887 :132人目の素数さん:2023/05/22(月) 04:37:51.94 ID:2UAex/JZ.net
Kに含まれる最小の体

888 :132人目の素数さん:2023/05/22(月) 04:38:50.53 ID:2UAex/JZ.net
標数pの体の素体ℤₚ

889 :132人目の素数さん:2023/05/22(月) 04:39:32.58 ID:2UAex/JZ.net
ℤₚ=𝔽ₚとする

890 :132人目の素数さん:2023/05/22(月) 04:41:16.07 ID:2UAex/JZ.net
有限体Kの0以外の元からなる乗法群K*

891 :132人目の素数さん:2023/05/22(月) 04:41:50.88 ID:2UAex/JZ.net
有限体K、乗法群K*

892 :132人目の素数さん:2023/05/22(月) 04:42:22.97 ID:2UAex/JZ.net
乗法群K*は巡回群である

893 :132人目の素数さん:2023/05/22(月) 04:42:58.37 ID:2UAex/JZ.net
生成元を原始根
という

894 :132人目の素数さん:2023/05/22(月) 04:43:42.40 ID:2UAex/JZ.net
部分体と

895 :132人目の素数さん:2023/05/22(月) 04:43:52.05 ID:2UAex/JZ.net
拡大体

896 :132人目の素数さん:2023/05/22(月) 04:45:59.06 ID:2UAex/JZ.net
KはF上のVector空間

897 :132人目の素数さん:2023/05/22(月) 04:47:32.43 ID:2UAex/JZ.net
dim_F K=[K: F]

898 :132人目の素数さん:2023/05/22(月) 04:49:41.51 ID:2UAex/JZ.net
[L: F]=[L: K][K: F]

899 :132人目の素数さん:2023/05/22(月) 04:50:56.24 ID:2UAex/JZ.net
αはK上代数的である

900 :132人目の素数さん:2023/05/22(月) 04:51:34.15 ID:2UAex/JZ.net
αはK上超越的である

901 :132人目の素数さん:2023/05/22(月) 04:52:25.29 ID:2UAex/JZ.net
αを根とする

902 :132人目の素数さん:2023/05/22(月) 04:52:38.21 ID:2UAex/JZ.net
次数最小の

903 :132人目の素数さん:2023/05/22(月) 04:52:47.75 ID:2UAex/JZ.net
モニックな

904 :132人目の素数さん:2023/05/22(月) 04:53:00.28 ID:2UAex/JZ.net
既約多項式

905 :132人目の素数さん:2023/05/22(月) 04:53:39.48 ID:2UAex/JZ.net
αのK上の最小多項式f(x)

906 :132人目の素数さん:2023/05/22(月) 04:54:52.42 ID:2UAex/JZ.net
αを根とする次数最小のモニックな既約多項式

907 :132人目の素数さん:2023/05/22(月) 04:55:46.75 ID:2UAex/JZ.net
LはK上代数的

908 :132人目の素数さん:2023/05/22(月) 04:56:48.65 ID:2UAex/JZ.net
Kにαを添加した体

909 :132人目の素数さん:2023/05/22(月) 04:57:05.64 ID:2UAex/JZ.net
K(α)

910 :132人目の素数さん:2023/05/22(月) 04:58:22.10 ID:2UAex/JZ.net
Kに添加した体

911 :132人目の素数さん:2023/05/22(月) 04:59:09.35 ID:2UAex/JZ.net
単純拡大

912 :132人目の素数さん:2023/05/22(月) 04:59:21.42 ID:2UAex/JZ.net
単拡大

913 :132人目の素数さん:2023/05/22(月) 05:08:50.46 ID:2UAex/JZ.net
K[]X→K[α]⊂L

914 :132人目の素数さん:2023/05/22(月) 05:09:18.87 ID:2UAex/JZ.net
X→α代入

915 :132人目の素数さん:2023/05/22(月) 05:09:46.69 ID:2UAex/JZ.net
これは環同型写像

916 :132人目の素数さん:2023/05/22(月) 05:10:27.28 ID:2UAex/JZ.net
K[α]の商体K(α)

917 :132人目の素数さん:2023/05/22(月) 05:11:12.53 ID:2UAex/JZ.net
K上の有理関数体と同型

918 :132人目の素数さん:2023/05/22(月) 05:11:28.53 ID:2UAex/JZ.net
K(x)

919 :132人目の素数さん:2023/05/22(月) 05:13:37.98 ID:2UAex/JZ.net
写像: K[ X]/(f)→K(α)

920 :132人目の素数さん:2023/05/22(月) 05:14:05.62 ID:2UAex/JZ.net
この場合、写像X'→α

921 :132人目の素数さん:2023/05/22(月) 05:15:18.43 ID:2UAex/JZ.net
同型写像でありK[α]は体になる

922 :132人目の素数さん:2023/05/22(月) 05:16:07.90 ID:2UAex/JZ.net
すなわちK[α]=K(α)

923 :132人目の素数さん:2023/05/22(月) 05:17:10.54 ID:2UAex/JZ.net
K⊂L、α∈Lとする

924 :132人目の素数さん:2023/05/22(月) 05:18:11.38 ID:2UAex/JZ.net
αを体K上代数的とする

925 :132人目の素数さん:2023/05/22(月) 05:19:21.97 ID:2UAex/JZ.net
[K(α): K]=n

926 :132人目の素数さん:2023/05/22(月) 05:20:37.67 ID:2UAex/JZ.net
f(x)のLにおける根

927 :132人目の素数さん:2023/05/22(月) 05:25:59.74 ID:2UAex/JZ.net
2つの有限体は元の数が等しければ互いに同型である

928 :132人目の素数さん:2023/05/22(月) 05:27:19.76 ID:2UAex/JZ.net
制限σ|F

929 :132人目の素数さん:2023/05/22(月) 05:27:48.19 ID:2UAex/JZ.net
自己同型写像

930 :132人目の素数さん:2023/05/22(月) 05:28:03.52 ID:2UAex/JZ.net
恒等写像

931 :132人目の素数さん:2023/05/22(月) 05:29:55.55 ID:2UAex/JZ.net
q=p^r、r≧1

932 :132人目の素数さん:2023/05/22(月) 12:18:01.92 ID:WcEyG/Gg.net
-(-a)=a

933 :132人目の素数さん:2023/05/22(月) 12:18:31.59 ID:WcEyG/Gg.net
0a=a0=0

934 :132人目の素数さん:2023/05/22(月) 12:19:46.40 ID:WcEyG/Gg.net
(-a)b=a(-b)=-ab

935 :132人目の素数さん:2023/05/22(月) 12:29:10.24 ID:WcEyG/Gg.net
a(b-c)=ab-ac

936 :132人目の素数さん:2023/05/22(月) 12:31:38.30 ID:WcEyG/Gg.net
(b-c)a=ba-ca

937 :132人目の素数さん:2023/05/22(月) 12:33:38.35 ID:WcEyG/Gg.net
三則が可能

938 :132人目の素数さん:2023/05/22(月) 12:34:07.40 ID:WcEyG/Gg.net
可換、簡約が可能とは限らない

939 :132人目の素数さん:2023/05/22(月) 13:00:59.24 ID:WcEyG/Gg.net
単位的環

940 :132人目の素数さん:2023/05/22(月) 13:02:54.39 ID:WcEyG/Gg.net
正則R*

941 :132人目の素数さん:2023/05/22(月) 13:08:47.72 ID:WcEyG/Gg.net
(ma)b=a(mb)=mab

942 :132人目の素数さん:2023/05/22(月) 23:21:50.44 ID:uY+yy519.net
零環{0}

943 :132人目の素数さん:2023/05/22(月) 23:22:44.76 ID:uY+yy519.net
0+0=0
0×0=0
よって0は単位元である

944 :132人目の素数さん:2023/05/22(月) 23:23:04.63 ID:uY+yy519.net
零環は0しかない

945 :132人目の素数さん:2023/05/22(月) 23:23:38.25 ID:uY+yy519.net
零元0

946 :132人目の素数さん:2023/05/22(月) 23:25:49.53 ID:uY+yy519.net
環R≠{0}
零環を除くとする

947 :132人目の素数さん:2023/05/22(月) 23:28:31.67 ID:uY+yy519.net
a≠0を元に持つ

948 :132人目の素数さん:2023/05/22(月) 23:29:30.58 ID:uY+yy519.net
0≠a=ae=a0=0
となり矛盾

949 :132人目の素数さん:2023/05/22(月) 23:29:46.32 ID:uY+yy519.net
よってe≠0

950 :132人目の素数さん:2023/05/22(月) 23:34:24.40 ID:uY+yy519.net
0+0=0と分配律により
∀a、a+0=aでありa=0とすると
0+0=0
(0+0)a=0a、0a+0a=0a
∴0a=0

951 :132人目の素数さん:2023/05/22(月) 23:35:08.61 ID:uY+yy519.net
a(0+0)=a0
a0+a0=a0
∴a0=0

952 :132人目の素数さん:2023/05/22(月) 23:35:33.88 ID:uY+yy519.net
よって0a=a0=0となる

953 :132人目の素数さん:2023/05/22(月) 23:36:50.84 ID:uY+yy519.net
-(-a)=a

954 :132人目の素数さん:2023/05/22(月) 23:37:59.13 ID:uY+yy519.net
(a⁻¹)⁻¹=a

955 :132人目の素数さん:2023/05/22(月) 23:39:24.43 ID:uY+yy519.net
群Gに置いて
aa⁻¹=e
これはaの逆元がa⁻¹
a⁻¹の逆元がaであることを示す

956 :132人目の素数さん:2023/05/22(月) 23:40:10.63 ID:uY+yy519.net
Aの逆元はA⁻¹であり
A=a⁻¹とすると
(a⁻¹)⁻¹=a

957 :132人目の素数さん:2023/05/22(月) 23:40:56.31 ID:uY+yy519.net
加法群において
a+(-a)=0

958 :132人目の素数さん:2023/05/22(月) 23:41:47.62 ID:uY+yy519.net
これはaの逆元が-a
-aの逆元がaであることを示す

959 :132人目の素数さん:2023/05/22(月) 23:42:31.10 ID:uY+yy519.net
Aの逆元は-A⇔表せるから
A=-aとして

960 :132人目の素数さん:2023/05/22(月) 23:43:00.41 ID:uY+yy519.net
-(-a)=aとなる

961 :132人目の素数さん:2023/05/22(月) 23:50:20.06 ID:uY+yy519.net
R*

962 :132人目の素数さん:2023/05/22(月) 23:51:33.59 ID:uY+yy519.net
a, b∈Rの中でa⁻¹、b⁻¹が存在するものだけを考える

963 :132人目の素数さん:2023/05/22(月) 23:52:48.42 ID:uY+yy519.net
a, b∈R*⇒a⁻¹, b⁻¹∈R*である

964 :132人目の素数さん:2023/05/22(月) 23:53:55.04 ID:uY+yy519.net
a⁻¹, b⁻¹∈R*⇒b⁻¹a⁻¹, a⁻¹b⁻¹∈R*である

965 :132人目の素数さん:2023/05/22(月) 23:54:39.08 ID:uY+yy519.net
するとab∈R*⇒b⁻¹a⁻¹∈R*となる

966 :132人目の素数さん:2023/05/22(月) 23:55:19.10 ID:uY+yy519.net
(ab)(b⁻¹a⁻¹)=eより逆元が存在する

967 :132人目の素数さん:2023/05/22(月) 23:56:05.15 ID:uY+yy519.net
ab∈R*より積に関してR*は閉じている

968 :132人目の素数さん:2023/05/22(月) 23:57:37.64 ID:uY+yy519.net
Rに関してae=ea=aでありR*に制限しても単位元eは同じものが同じ役割を持つ。すなわち単位元は存在する。

969 :132人目の素数さん:2023/05/22(月) 23:58:49.01 ID:uY+yy519.net
結合律もRで成り立つのでR*に制限しても成り立つ

970 :132人目の素数さん:2023/05/23(火) 00:04:20.23 ID:wmZr8viD.net
Rに関して0を除外した場合

971 :132人目の素数さん:2023/05/23(火) 00:04:46.61 ID:wmZr8viD.net
R*は群をなすかどうか

972 :132人目の素数さん:2023/05/23(火) 00:05:38.95 ID:wmZr8viD.net
結果的に群をなす

973 :132人目の素数さん:2023/05/23(火) 00:05:51.06 ID:wmZr8viD.net
ことがわかった

974 :132人目の素数さん:2023/05/23(火) 00:06:10.63 ID:wmZr8viD.net
積に関して閉じている

975 :132人目の素数さん:2023/05/23(火) 00:06:38.66 ID:wmZr8viD.net
単位元の存在

976 :132人目の素数さん:2023/05/23(火) 00:06:52.84 ID:wmZr8viD.net
逆元の存在

977 :132人目の素数さん:2023/05/23(火) 00:07:08.98 ID:wmZr8viD.net
結合律の成立

978 :132人目の素数さん:2023/05/23(火) 00:07:41.93 ID:wmZr8viD.net
逆元を持つ元のみの集合

979 :132人目の素数さん:2023/05/23(火) 00:08:03.49 ID:wmZr8viD.net
結局0が除外サれるだけ

980 :132人目の素数さん:2023/05/23(火) 00:08:24.77 ID:wmZr8viD.net
0には逆元は存在しない

981 :132人目の素数さん:2023/05/23(火) 00:08:52.26 ID:wmZr8viD.net
加法群としてならば存在する

982 :132人目の素数さん:2023/05/23(火) 00:10:13.42 ID:wmZr8viD.net
R*=R-{0}

983 :132人目の素数さん:2023/05/23(火) 00:10:38.04 ID:wmZr8viD.net
R\{0}

984 :132人目の素数さん:2023/05/23(火) 00:12:05.63 ID:wmZr8viD.net
R=R*と仮定すると

985 :132人目の素数さん:2023/05/23(火) 00:12:38.12 ID:wmZr8viD.net
0∈Rより0∈R*となる

986 :132人目の素数さん:2023/05/23(火) 00:13:14.73 ID:wmZr8viD.net
∀a∈R*、a⁻¹が存在するから

987 :132人目の素数さん:2023/05/23(火) 00:13:27.77 ID:wmZr8viD.net
a=0として

988 :132人目の素数さん:2023/05/23(火) 00:13:46.12 ID:wmZr8viD.net
0×0⁻¹=e

989 :132人目の素数さん:2023/05/23(火) 00:14:02.50 ID:wmZr8viD.net
∀a、0a=0より

990 :132人目の素数さん:2023/05/23(火) 00:14:21.28 ID:wmZr8viD.net
0=eとかり不合理

991 :132人目の素数さん:2023/05/23(火) 00:14:48.61 ID:wmZr8viD.net
よってR≠R*である

992 :132人目の素数さん:2023/05/23(火) 00:15:09.66 ID:wmZr8viD.net
あり得ない物を持ち込んで

993 :132人目の素数さん:2023/05/23(火) 00:15:31.62 ID:wmZr8viD.net
矛盾を導く

994 :132人目の素数さん:2023/05/23(火) 00:22:57.43 ID:wmZr8viD.net
分配律を利用する

995 :132人目の素数さん:2023/05/23(火) 00:24:14.71 ID:wmZr8viD.net
(-a)b+ab=(-a+a)b=0b=0
∴(-a)b=-ab

996 :132人目の素数さん:2023/05/23(火) 00:25:31.12 ID:wmZr8viD.net
a(-b)+ab=a(-b+b)=a0=0
∴a(-b)=-ab

997 :132人目の素数さん:2023/05/23(火) 00:28:00.31 ID:wmZr8viD.net
よってA(-b)=-Ab
A=-aとして
-(-a)(-b)=-(-a)b=ab

998 :132人目の素数さん:2023/05/23(火) 00:29:55.29 ID:wmZr8viD.net
分配律より

999 :132人目の素数さん:2023/05/23(火) 00:30:32.40 ID:wmZr8viD.net
a(b-c)=

1000 :132人目の素数さん:2023/05/23(火) 00:30:51.82 ID:wmZr8viD.net
分配律により

1001 :132人目の素数さん:2023/05/23(火) 00:32:12.80 ID:wmZr8viD.net
a(b+(-c))=ab+a(-c)=ab-ac

1002 :132人目の素数さん:2023/05/23(火) 00:34:06.82 ID:wmZr8viD.net
-a=+(-a)

1003 :132人目の素数さん:2023/05/23(火) 00:34:37.42 ID:wmZr8viD.net
(b-c)a=

1004 :132人目の素数さん:2023/05/23(火) 00:35:03.92 ID:wmZr8viD.net
(b+(-c))a=

1005 :132人目の素数さん:2023/05/23(火) 00:35:30.31 ID:wmZr8viD.net
bc+(-c)a=

1006 :132人目の素数さん:2023/05/23(火) 00:36:03.53 ID:wmZr8viD.net
ba-ca

1007 :2ch.net投稿限界:Over 1000 Thread
2ch.netからのレス数が1000に到達しました。

総レス数 1007
345 KB
掲示板に戻る 全部 前100 次100 最新50
read.cgi ver.24052200